Taxation Quizzer

  • Uploaded by: awitchie
  • 0
  • 0
  • February 2021
  • PDF

This document was uploaded by user and they confirmed that they have the permission to share it. If you are author or own the copyright of this book, please report to us by using this DMCA report form. Report DMCA


Overview

Download & View Taxation Quizzer as PDF for free.

More details

  • Words: 18,711
  • Pages: 61
Loading documents preview...
TAXATION QUIZZER PART 1 BASIC PRINCIPLES OF TAXATION 1.

Which theory in taxation states that without taxes, a government would be paralyzed for lack of power to activate and operate it, resulting in its destruction? a. Power to destroy theory b. Lifeblood theory c. Sumptuary theory d. Symbiotic doctrine

2.

The actual effort exerted by the government to effect the exaction of what is due from the taxpayer is known as a. Assessment. b. Levy. c. Payment. d. Collection.

3.

Although the power of taxation is basically character, it is NOT the function of Congress to a. Fix with certainty the amount of taxes. b. Collect the tax levied under the law. c. Identify who should collect the tax. d. Determine who should be subject to the tax.

4.

An example of a tax where the concept of progressivity finds application is the a. Income tax on individuals. b. Excise tax on petroleum products. c. Value-added tax on certain articles. d. Amusement tax on boxing exhibitions.

5.

Ligaya Educational Foundation, Inc., a stock educational institution organized for profit, decided to lease for commercial use a 1,500 sq. m. portion of its school. The school actually, directly, and exclusively use the rents for the maintenance of its school buildings, including payment of janitorial services. Is the leased portion subject to real property tax? a. Yes, since Lualhati is a stock and for profit educational institution. b. No, since the school actually, directly, and exclusively used the rents for educational purposes. c. No, but it may be subject to income taxation on the rents it receives. d. Yes, since the leased portion is not actually, directly, and exclusively used for educational purposes.

6.

Which among the following concepts of taxation is the basis for the situs of income taxation? a. Lifeblood doctrine of taxation b. Symbiotic relation in taxation

legislative

in

c. d.

7.

Compensatory purpose of taxation Sumptuary purpose of taxation

DONOR’S TAX Which of the following transactions is deemed a taxable gift? a. Condonation or remission of a debt b. Sale of residential house and lot for less than adequate and full consideration in money or money’s worth c. Both (a) and (b) d. Neither (a) nor (b)

8.

Which of the following statements relative to donor’s tax is false? a. The spouses shall file separate donor’s tax returns where the thing donated is common property. b. Each parent shall be entitled to the P10,000 exemption on account of marriage of a child. c. Exemptions and deductions cannot be claimed where the 30% tax rate on stranger is applicable. d. None of the foregoing.

9.

The spouses Esme and Carlisle wanted to donate a parcel of land to their son Edward who is getting married in December, 2016. The parcel of land has a zonal valuation of P420,000.00. What is the most efficient mode of donating the property? a. The spouses should first donate in 2016 a portion of the property valued at P20,000, then spread the P400,000 equally for 2017, 2018, 2019 and 2020. b. Spread the donation over a period of 5 years by the spouses donating P100,000 each year from 2016 to 2020. c. The spouses should each donate a P110,000 portion of the value of the property in 2016 then each should donate P100,000 in 2017. d. The spouses should each donate a P100,000 portion of the value of the property in 2016, and another P100,000 each in 2017. Then, in 2018, Esme should donate the remaining P20,000.

10.

Exempted from donor’s taxation are gifts made a. For the use of the barangay. b. In consideration of marriage. c. To a school which is a stock corporation. d. To a for-profit government corporation.

11.

Caroline donated P110,000.00 to her friend Vicky who was getting married. Caroline gave no other gift during the calendar year. What is the donor's tax implication on Caroline’s donation? a. The P100,000 portion of the donation is exempt since given in consideration of marriage. b. A P10,000 portion of the donation is exempt being a donation in consideration of marriage.

c. d.

Caroline shall pay a 30% donor's tax on the P110,000 donation. The P100,000.00 portion of the donation is exempt under the rate schedule for donor's tax.

12.

A non-stock, non-profit school always had cash flow problems, resulting in failure to recruit well-trained administrative personnel to effectively manage the school. In 2017, Don Leon donated P100 million pesos to the school, provided the money shall be used solely for paying the salaries, wages, and benefits of administrative personnel. The donation represents less than 10% of Don Leon's taxable income for the year. Is he subject to donor's taxes? a. No, since the donation is actually, directly, and exclusively used for educational purposes. b. Yes, because the donation is to be wholly used for administration purposes. c. Yes, since he did not obtain the requisite NGO certification before he made the donation. d. No, because the donation does not exceed 10% of his taxable income for 2017.

13.

What law shall govern the imposition of donor’s tax? a. The law in force at the time of perfection of the donation b. The law in force at the time of completion of the donation c. The law in force at the time of perfection or completion depending upon the agreement of the parties d. None of the choices

14.

Andy, married, donated a land commonly owned by him and her spouse worth P500,000 to her friend Joan. Only Andy signed the deed of donation. Joan assumed P200,000 unpaid mortgage on the property. How much is the donor’s tax due? a. P6,000 c. P1,000 b. P90,000 d. P45,000 500k - 200k = 300k x 30% = 90,000

15.

Dondie, resident citizen, made the following donations on April of the current year:  To his sister, Donna,P175,000 worth of property situated Paris, France. The donor’s tax paid is P40,000.  To Dara, his girlfriend in the Philippines, jewelry valued P225,000.  To International Rice Research Institute, cash amounting P50,000. The donor’s tax due after tax credit is – a. P69,000 c. P29,000 b. P38,813 d. P5,625

28 in at to

225,000 x 30% = 67,500 + (175,000 - 100,000 (69,000 x 175k/400k) = 38,813

x 2% Tabular) = 69,000 -

ESTATE TAX 16. Which of the following is not a part of the gross estate? a. Conjugal property b. Community property c. Exclusive property of the decedent d. Exclusive property of the surviving spouse 17.

Who among the following transferors is not liable for estate tax on the property transferred during his lifetime? a. The testator who bequeaths property to his heirs in a last will and testament executed and probated during his lifetime b. The donor who reserves his right to amend or revoke the donation of property in favor of the donee c. The donee of an appointed property who is required under a power of appointment to transfer such property upon death to his eldest child d. The transferor of personal property who sold it for insufficient consideration

18.

Which of the following properties of the spouses will be part of common properties under a regime of Conjugal Partnership of Gains? a. Land inherited during the marriage b. Fruits of land inherited c. Jewelry inherited during the marriage d. Building donated before marriage

19.

Which of the following items is not considered as a “special deduction” in computing the taxable net estate of the decedent? a. Vanishing deduction b. Medical expenses c. Standard deduction d. Family home allowance

20.

When the payment of estate tax will cause undue hardship upon the heirs or the estate which is undergoing judicial settlement before the court, the BIR Commissioner may grant an extension for a period not exceeding: a. 5 years b. 3 years c. 2 years d. 1 year

Next three (3) questions are based on the following: Orland, married resident citizen, died on August 20, 2016. The estate reported the following assets and deductions: Conjugal Properties: Fishpond, Bulacan P1,500,000 Family Home, Makati 1,500,000 Cash in bank 900,000

Exclusive Properties of Orland: Land, inherited from his father who died on July 20, 2012. The value of land at the time of inheritance was P210,000. The land was mortgaged for P30,000 which was unpaid at the time of death of his father,P10,000 of which was paid by Orland before he died. Land, donated on February 14, 2013 by his mother who died on November 2, 2014. The value of the land when donated was P500,000 while upon death of his mother was P400,000. Exclusive Properties of Wife: Farm in Laguna, acquired before marriage

22.

23.

The gross estate is: a. P4,000,000 b. P4,710,000

600,000

2,000,000

Deductions claimed: Funeral expenses Fire loss of apartment (occurred 4 months after death) Bad debts (represents unpaid receivable from Bert, an insolvent) Mortgage on inherited land Vanishing deduction on inherited land Vanishing deduction on donated land Standard deduction 21.

P400,000

c. d.

P7,000,000 P5,000,000

The vanishing deduction is: a. P184,000 b. P220,800

c. d.

The net taxable estate is: a. P819,200 b. P804,200

c. P829,200 d. P579,200

P255,760 P292,560

250,000 80,000 100,000 30,000 40,000 20,000 2,000,000

SOLUTION: Properties Exclusive Fishpond, Bulacan Family Home, Makati Cash in bank Land, inherited from Father died on July 20, 2012. 400,000 Land, Donated from Mother on Feb 14, 2013. 600,000 Claims to Insolvent GROSS ESTATE 1,000,000 Allowable Deductions: Funeral Expenses Fire Loss Bad Debts Mortgage on Land (20,000) Vanishing Deductions* (220,800) Net Estate before Special Deductions 759,200 Special Deductions: Family Home (1,500,000 x 1/2) Special Deduction Share of Surviving Spouse NET TAXABLE ESTATE

Conjugal P1,500,000 1,500,000 900,000

Total 1,500,000 1,500,000 900,000 400,000

100,000 4,000,000 (200,000) (80,000) (100,000)

3,620,000

*VANISHING DEDUCTIONS: Land Inherited by Father: Value at the time of Death Less: Mortgage Paid Initial Basis Pro rata: 200/5000 x 400,000 Final Basis Vanishing Rate (4 years but not more 5 yrs) Vanishing Deduction

600,000 100,000 5,000,000 (200,000) (80,000) (100,000) (20,000) (220,800) 4,379,200 (750,000) (1,000,000) (1,810,000) 819,200

210,000 (10,000) 200,000 (16,000) 184,000 20% 36,800

Land Donated by Mother: Value at the time of Donation Initial Basis Pro rata: 500/5000 x 400,000 Final Basis Vanishing Rate (3 years but not more 4 yrs) Vanishing Deduction

500,000 500,000 (40,000) 460,000 x 40% 184,000

TOTAL VANISHING DEDUCTIONS

220,800

**If TRAIN Law is applied the Estate tax is P 49,152 (P819,200 x 6%)

24.

Abe, married resident alien, died on January 15, 2017. She left the following properties, expenses and obligations: Community properties, Philippines (including family home valued at P1,800,000) Community properties, Abroad Exclusive properties, Philippines Actual funeral expenses Judicial expenses Medical expenses (incurred w/in 1yr. before death) Devise to National Gov’t Legacy to Local Gov’t The net taxable estate is: a. P3,780,000 b. P3,680,000

RESIDENT ALIEN Particulars Exclusive All Properties w/i & w/o 3,000,000 Funeral Expense Judicial Expense Transfers (50,000 + 70,000) (120,000) Gross Estate 2,880,000 Share of Surviving Spouse Medical Expenses Family Home (1/2 of 1,800,000) Standard Deductions Net Estate 25.

2,000,000 3,000,000 300,000 200,000 600,000 50,000 70,000

P3,580,000 P3,530,000 Community 7,000,000 (200,000) (200,000) 6,600,000

Total 10,000,000 (200,000) (200,000) (120,000) 9,480,000 (3,300,000) (500,000) (900,000) (1,000,000) 3,780,000

Based on the above problem, if the decedent is a non-resident alien, how much is the net taxable estate? a. P2,755,000 c. P5,220,000 b. P2,880,000 d. P5,380,000

NON - RESIDENT ALIEN Particulars Exclusive All Properties w/i Only 3,000,000 Funeral Expense 200,000 Judicial Expense 200,000 Allowable Deduction 400,000 x 8M/10M Transfers(50,000 + 70,000) (120,000) Gross Estate 2,880,000 Share of Surviving Spouse Net Estate 26.

c. d.

P5,000,000

Community 5,000,000

Total 8,000,000

(320,000)

(320,000) (120,000) 7,560,000 (2,340,000) 5,220,000

4,680,000

Arthur, Filipino, married died leaving the following estate:

Car acquired before marriage by Arthur

P 300,000

Car acquired before marriage by wife House and lot acquired during marriage Jewelries of wife Personal properties inherited by Arthur during marriage Benefits from SSS Retirement benefits Proceeds of group insurance taken by his employer Land inherited by the wife during marriage Income earned from the land inherited by wife (25% of which was earned after death)

450,000 1,500,000 100,000 250,000 50,000 150,000 75,000 1,000,000 200,000

16. How much is the gross estate if the property relationship is conjugal partnership of gains is: a. P2,600,000 c. P1,950,000 b. P3,600,000 d. P2,200,000 Car acquired before marriage by Arthur House and lot acquired during marriage Personal properties inherited by Arthur during marriage Income earned from the land inherited by wife (25% of which was earned after death) Gross Estate 27. a. b.

P 300,000 1,500,000 250,000 150,000 2,200,000

Based on the preceding number, the gross estate if the property relationship is absolute community of property is: P2,600,000 c. P1,950,000 P3,600,000 d. P2,500,000

Car acquired before marriage by Arthur Car acquired before marriage by wife House and lot acquired during marriage Jewelries of wife Personal properties inherited by Arthur during marriage Gross Estate

P 300,000 450,000 1,500,000 100,000 250,000 2,600,000

VAT 28.

LBJ made the following sales during the 12-month period: Sales, VAT taxable transactions Sales, VAT zero-rated transactions Sales, VAT exempt transactions Total

P1,500,000 400,000 100,000 P2,000,000

Which of the following statements is correct? a. LBJ may not register under the VAT system because his sales from VAT taxable transactions did not exceed P1,919,500. b. LBJ may not register under the VAT system because his sales from VAT taxable and zero-rated transactions did not exceed P1,919,500.

c. LBJ is required to register because his total 12-month sales exceeded P1,919,500. d. None of the foregoing. 29.

Which of the following a. Common carriers Philippines b. Common carriers Philippines Common carriers c. Philippines d. Common carriers Philippines

is exempt from VAT? transporting passengers by air within the transporting passengers by sea within the transporting passengers by land within the transporting

cargoes

by

air

within

the

30.

Which statement is correct about value-added tax on goods or properties sold? a. It is based on gross sales and not on net sales; May be due even if the goods or properties were not actually b. sold; c. Does not cover goods exported; d. It forms part of the selling expense of the trader.

31.

For value-added tax purposes, which of the following transactions of a VAT-registered taxpayer may not be zero-rated? a. Export sales b. Foreign currency denominated sales c. Sale of goods to the Asian Development Bank d. Sale of goods to an export oriented enterprise

32.

A subdivision developer sold five (5) residential house and lots, each to different vendees, for P3,000,000 per lot, or a total sales of P15,000,000 for the taxable period. These sales shall be classified as: a. 12% VAT transactions b. 0% VAT transactions c. VAT exempt transactions d. None of the foregoing **3,199,000 each is the threshold

33.

CP operated a retail business that had been generating sales not exceeding the threshold for VAT exempt persons. However, he desires to be registered under the VAT system for the first time in order to benefit from input tax credits. What benefit may CP be entitled to once he registers under the VAT system? a. Tax refund b. Presumptive input tax credit c. Transitional input tax credit d. None of the foregoing

34.

What institution is required to deduct and withhold a final VAT of 5% on the purchase of goods or services subject to VAT? a. National government or any political subdivision thereof b. Government-owned or controlled corporations c. Both (a) and (b) d. Neither (a) nor (b)

35.

In the value-added tax on sale of services, the output tax is computed: a. On the billings of the month b. On collections of the month on all billings made c. On the contract price of contracts completed during the taxable period d. Only and strictly on labor performed under the contract for services

36.

Which statement is wrong? a. There is a transitional input tax from purchases of goods properties; There is a transitional input tax from purchases b. services; c. There is a transitional input tax from purchases materials; d. There is a transitional input tax from purchases supplies.

or of of of

37.

Which of the following statements is correct on the inventory balance in the financial statements at any given date of a VATregistered person? Balance, net of input taxes a. b. Balance, inclusive of input taxes c. Balance on which the transitional input tax is computed annually d. Balance where the VAT thereon may be calculated by multiplying it by 12%

38.

Genson Distribution Inc., a VAT taxpayer, had the following data in a month: Cash sales Open account sales Consignment: 0 to 30 days old (on which there were remittances from consignees of P200,000) 31 to 60 days old 61 days old and above How much is the output tax? a. P348,000 c. P264,000 b. P216,000 d. P108,000

P200,000 500,000

600,000 700,000 900,000

Cash sales Open account sales Consignment: 0 to 30 days old (on which there were remittances from consignees of P200,000) 61 days old and above Total VATABLE SALES VAT RATE OUTPUT VAT 42.

P200,000 500,000

200,000 900,000 1,800,000 12% 216,000

The financial records of Benz Corp., a VAT-registered taxpayer, for the taxable year 2016 disclosed the following: Local sales to private entities 1,500,000 Export Sales 500,000 Local sales to government 800,000 How much is the total sales subject to value-added tax? a. P2,800,000 c. P2,000,000 b. P2,300,000 d. P1,500,000 Local sales to private entities Export Sales Local sales to government Total VATABLE SALES

43.

1,500,000 500,000 800,000 2,800,000

Mantika Corp., a VAT-registered Corp., is a producer of cooking oil from coconut and corn. It had the following data for the month of January 2017: Sales, gross of VAT P 784,000 Corn & Coconut, 12-31-16 50,000 Purchases of Corn & Coconut 330,000 Corn & Coconut, 1-31-17 20,000 Purchases from VAT suppliers, VAT included: Packaging Materials 56,000 Supplies 16,800 The value-added tax payable for the month: a. P56,060 c. P60,650 b. P54,900 d. P63,000 Sales, gross of VAT P 784,000 Output TAX 84,000 Purchases of Corn & Coconut (330,000 x4%) Purchases from VAT suppliers, VAT included: Packaging Materials 56,000 Supplies 16,800 72,800 x3/28 VAT PAYABLE

(13,200)

(7,800) 63,000

44.

Bunga Inc., a VAT taxpayer, is engage in the business of processing of fruits. Its data on sales and purchases for the month of August are provided below: Sales P200,000 Purchases: Fresh Fruits 30,000 Raw sugarcane 12,000 Tin Can, gross of VAT 12,320 Paper Labels, net of VAT 5,000 Cardboard for boxes, net of VAT 8,000 Freight, gross of VAT (50% still unpaid) 10,080 How much is the value-added tax payable? a. P20,580 c. P19,380 b. P18,900 d. P20,100

Sales Purchases: Fresh Fruits Raw sugarcane Tin Can, gross of VAT Paper Labels, net of VAT Cardboard for boxes, net of VAT Freight, gross of VAT (50% still unpaid) VAT PAYABLE 45.

P200,000

Output Tax

30,000 12,000 12,320 5,000 8,000

24,000

Input Tax Input Tax Input Tax

10,080

(1,320) ( 600) ( 960)

Input Tax

( 540) 20,580

Bahay Kubo Inc. is a real estate dealer. Details of its sales during the year showed the following: Date of sale June 2, 2017 Consideration in the deed of sale P 5,000,000 Fair market value in the assessment rolls 4,800,000 Zonal Value 5,200,000 Schedule of payments: June 2, 2017 1,000,000 June 2, 2018 2,000,000 June 2, 2019 2,000,000 How much is the output tax to be recognized for the June 2, 2018 payment? c. P249,600 a. P0 b. P124,800 d. P624,000 **Zonal Value 5,200,000 249,600 Output Tax for 2018

46.

x

12%

=

624,000

x

2M/5M

=

Assuming that the scheduled payment on June 2, 2017 is P2,000,000, how much is the output tax to be recognized for the June 2, 2019 payment? a. P0 c. P249,600 b. P124,800 d. P624,000

Zero as in 0 for the sale will no longer qualify as Installment Sales. 47.

Mr. Karpentero, a vat-registered building contractor, has the following data on gross receipts in a month, any tax not included: 

From Mr. A, a private property owner, final payment on the contract price, net of 5% agreed retention fee of P2,850,000



From Mr. B, a payment of 5% retention on the contract price previously made by him P100,000



From Mr. C, for materials used in the construction 500,000

How much is the output tax? a. P414,000 b. P342,000 Final Payment on Contracts Retention Materials TOTAL Receipts VAT Rate Output VAT 48.

c. d.

P72,000 P62,000 2,850,000 100,000 500,000 3,450,000 12% 414,000

COC Inc., in its first month of operation, and as a VAT taxpayer, purchased various fixed assets. Purchases of fixed assets in the first month were as follows: Light equipment, with a useful life of 3 years Heavy equipment, with a useful life of 10 years

P

300,000 4,000,000

How much is the input tax available for the month? a. P516,000 c. P480,000 b. P9,000 d. P8,600 Light Equipment Heavy Equipment Total Input VAT 49.

300,000/36 x 12% 4,000,000/60 x 12%

1,000 8,000 9,000

Kusina Co., had its kitchen assembled by a VAT taxpayer. It took six months for the contractor to finish the work. Kusina Co. purchased materials in July from VAT suppliers at a cost of P500,000, VAT not included. Payment to the contractor in July 2017 on the Construction in Progress, VAT not included was: On contractor’s billing in June On contractor’s billing in July

P100,000 70,000

The input tax available in July is: a. P0 c. b. P80,400 d.

P60,000 P20,400

Materials from VAT Supplier On contractor’s billing in June On contractor’s billing in July Total VAT Rate Output Tax 50.

500,000 100,000 70,000 670,000 12% 80,400

Data from the books of accounts of a VAT taxpayer for February: Domestic Exports Sales P 2,000,000 8,000,000 Purchases: From VAT Suppliers: Goods for sale 600,000 2,400,000 Supplies & services 90,000 360,000 From Suppliers Paying percentage tax: Goods for sale 100,000 1,500,000 Supplies & services 20,000 80,000 If the input taxes attributable to zero-rated sales are claimed as tax credit, the net value-added tax refundable is: a. P136,000 c. P145,000 b. P203,924.70 d. P174,000 Output Tax (2,000,000 x 12%) Input Tax on Domestic Sales (690,000 x 12%) Input Tax on Zero Rated (2,760,000 x 12%) Refundable VAT

240,000 (82,800) (331,200) (174,000)

INCOME TAXATION 51. C. Lee, Chinese national, arrived in the Philippines on January 1, 2012 to visit his Filipina paramour. He planned to stay in the country until December 31, 2016, by which time he would go back to his legal wife and family in China. C. Lee derived income during his stay here in the Philippines. For the taxable year 2012, C. Lee shall be classified as a: a. Resident alien b. Non-resident alien engaged in trade or business in the Philippines c. Non-resident alien not engaged in trade or business in the Philippines d. Special alien employee 52.

The following individual taxpayers are subject income tax rates of 5%-32%, except a. Filipino citizens b. Resident aliens c. Non-resident alien engaged in trade or Philippines

to the graduated

business

in

the

d.

Non-resident alien not engaged in trade or business in the Philippines

53.

In which of the following cases will the dividend income from a foreign corporation be classified as “income without” a. Less than 50% of the foreign company’s gross income for the preceding three (3) years prior to the dividend declaration was derived from sources within the Philippines. b. 50% of the foreign company’s gross income for the preceding three (3) years prior to the dividend declaration was derived from sources within the Philippines. c. More than 50% of the foreign company’s gross income for the preceding three (3) years prior to the dividend declaration was derived from sources within the Philippines. d. Always classified as income without”.

54.

D’ Lion, Inc., a Philippine corporation, sold through the local stock exchange 10,000 PLDT shares that it bought 2 years ago. D’ Lion sold the shares for P2 million and realized a net gain of P200,000. How shall it pay tax on the transaction? a. It shall declare a P2 million gross income in its income tax return, deducting its cost of acquisition as an expense. b. It shall report the P200,000 in its corporate income tax return adjusted by the holding period. c. It shall pay 5% tax on the first P100,000 of the P200,000 and 10% tax on the remaining P100,000. d. It shall pay a tax of one-half of 1% of the P2 million gross sales.

55.

The payor of passive income subject to final tax is required to withhold the tax from the payment due the recipient. The withholding of the tax has the effect of A final settlement of the tax liability on the income. a. b. A credit from the recipient's income tax liability. c. Consummating the transaction resulting in an income. d. A deduction in the recipient's income tax return.

56.

Winterfell, Inc., bought a parcel of land in 2015 for P7 million as part of its inventory of real properties. In 2017, it sold the land for P12 million which was its zonal valuation. In the same year, it incurred a loss of P6 million for selling another parcel of land in its inventory. These were the only transactions it had in its real estate business. Which of the following is the applicable tax treatment? a. Winterfell shall be subject to a tax of 6% of P12 million. b. Winterfell could deduct its P6 million loss from its P5 million gain. c. Winterfell’s gain of P5 million shall be subject to the holding period. d. Winterfell's P6 million loss could not be deducted from its P5 million gain.

57.

Passive income includes income derived from an activity in which the earner does not have any substantial participation. This type of income is a. Usually subject to a final tax. b. Exempt from income taxation. c. Taxable only if earned by a citizen. d. Included in the income tax return.

58.

In 2017, Alice earned P500,000 as income from her beauty parlor and received P250,000 as Christmas gift from her aunt. She had no other receipts for the year. She spent P150,000 for the operation of her beauty parlor. For tax purposes, her gross income for 2017 is a. P750,000 c. P350,000 b. P500,000 d. P600,000

59.

Which of the following items is not part of gross income to be reported in the income tax return? a. Increase in value of land b. Gambling winnings c. Prize of P10,000 d. Gain from sale of store’s air conditioner

60.

Mr. Yu leased his lot to Mr. Uy. The contract calls for Mr. Uy to construct a house which would serve as the residence of the latter, the ownership thereof to be vested in Mr. Yu after the expiration of the lease. When the house was completely constructed, the remaining term of the lease was 10 years. The residential house had an estimated useful life of 15 years. What is the tax implication of the leasehold improvement? a. Mr. Yu derives taxable income on the improvement; Mr. Uy can claim depreciation expense as a deduction from gross income. Mr. Yu derives taxable income on the improvement; Mr. Uy b. cannot claim depreciation expense as a deduction from gross income. c. Mr. Yu does not derive taxable income on the improvement; Mr. Uy cannot claim depreciation expense as a deduction from gross income. d. Mr. Yu does not derive taxable income on the improvement; Mr. Uy can claim depreciation expense as a deduction from gross income.

61.

Assume the same facts in the immediately preceding number, except that at the time of the completion of the residential house, the remaining term of the lease was 15 years while the useful life of the house was 10 years. What is the tax implication of the leasehold improvement? a. Mr. Yu derives taxable income on the improvement; Mr. Uy can claim depreciation expense as a deduction from gross income. b. Mr. Yu derives taxable income on the improvement; Mr. Uy cannot claim depreciation expense as a deduction from gross income.

c.

d.

Mr. Yu does not derive taxable income on the improvement; Mr. Uy cannot claim depreciation expense as a deduction from gross income. Mr. Yu does not derive taxable income on the improvement; Mr. Uy can claim depreciation expense as a deduction from gross income.

62.

Which of the following expenses may be deducted from gross compensation income? a. Depreciation of permanent assets b. Premium payments on health and/or hospitalization insurance c. Bad debts written off d. Optional standard deduction

63.

Which of the following items of interest expense may be deducted from gross income? a. Interest on corporation’s preferred stock b. Interest on loan for construction of a rest house Interest for delinquency in the payment of percentage tax c. d. Interest on bank loan to finance petroleum exploration

64.

Which a. b. c. d.

65.

Who among the following taxpayers may not claim a tax credit or deduction on income tax paid to foreign countries? a. Resident citizens Resident aliens b. c. Domestic corporations d. General Co-Partnerships

66.

The loss from sale or exchange of property is deductible from gross income where the sale or exchange is: a. Between fiduciary of a trust and the fiduciary of another trust if they have the same grantor b. Between fiduciary of a trust and the beneficiary of such trust c. Between an individual and his first cousin d. Between an individual and a corporation if the former owns more than 50% in value of the outstanding capital stock of the latter

67.

Anne, claimed a bad debt of P50,000 as a deductible expense in the taxable year 2016. In 2017, Anne was able to recover the P50,000 debt already written off in the preceding year. What is the treatment for tax purposes of the recovery of the bad debt? a. Report the recovery of the bad debt as gross income in 2016. b. Report the recovery of the bad debt as gross income in 2017.

of the following taxes may be deducted from gross income? Percentage tax on sale of listed stock Business permit fee paid to the city government Income tax Tax on interest on bank deposit

c. d.

Disregard the recovery of the bad debt. Amend the 2016 income tax return to rectify the deduction for bad debt claimed.

68.

Which a. b. c. d.

of the following assets shall be subject to depletion? Machinery Land containing ore deposit Commercial Goodwill

69.

Which of the following individual taxpayers may claim basic and additional personal exemptions for income tax purposes? a. Non-resident aliens engaged in trade or business in the Philippines, in the absence of reciprocity b. Non-resident aliens not engaged in trade or business in the Philippines c. Both (a) and (b) Neither (a) nor (b) d.

70.

Harry works as financial consultant in an oil firm in Dubai. Aside from his salary thereat, he also maintains a 10-door apartment in Manila which he inherited from his parents when he was already married. On the other hand, Wilma, his wife, is employed as a loan officer at a local bank. Data pertaining to their dependents appear below for the taxable year 2017:     

Anton - Son who turned 23 on April 1, 2017; incapable of self-support due to loss of both legs in an accident; Bunny - 21 year old daughter who is taking up culinary arts in Paris, France; Charlie - 15-year old adulterous son of Harry living with the couple; Dina - 12-year old child who died from a vehicular accident on January 1, 2017; and Evan - 80-year old father of Wilma, supported by her and living with the couple.

The basic and additional personal exemptions of Harry for the taxable year 2017 amounts to: P50,000 and P100,000, respectively a. b. P50,000 and P75,000, respectively c. P50,000 and P0, respectively d. P0 and P0, respectively 71.

Assume the same facts above, the basic and additional personal exemptions of Wilma for the taxable year 2016 amount to: a. P50,000 and P100,000, respectively b. P50,000 and P75,000, respectively c. P50,000 and P0, respectively d. P0 and P0, respectively

72.

Which of asset? a. It b. It c. It d. It

the following statements does not characterize a capital may be real or personal property. is not always subject to a holding period. is normally subject to value-added tax when it is sold. is not always subject to a final tax.

73.

Which of the following transactions is exempt from capital gains tax? a. The sale of the principal residence of the taxpayer where the entire proceeds is used to purchase a vacation lot at Tagaytay b. The sale of a beach lot of the taxpayer where the entire proceeds is used to construct his principal residence c. The sale of the principal residence of the taxpayer for the second time in ten (10) years to purchase another principal residence d. None of the choices

74.

Which of the following transactions is treated as a capital asset transaction for income tax purposes? a. Sale of a residential lot by a subdivision developer b. Sale of a used delivery truck by a retailing company c. Liquidation of partnership business d. Sale of shares of stock by a dealer in securities

75.

Which of the following is not an attribute of a deferred-payment sale? a. The initial payments exceed 25% of the selling price in the year of sale. b. The obligations or promissory notes received by the vendor from the vendee are considered as equivalent to cash. The tax may be paid in installments. c. d. The sale involves both real and personal property.

76.

The deductible expenses of an estate may consist of: a. Deductible expenses allowed to an individual taxpayer b. Income distributed to beneficiaries c. Both (a) and (b) d. Neither (a) nor (b)

77.

Determine which of the following trusts shall the taxable income be consolidated and the income tax thereon computed on the basis of such consolidated income? a. Trust No. 1 and Trust No. 2 have the same grantor and with different beneficiaries. b. Trust No. 1 and Trust No. 2 have the same grantor and the same beneficiary. c. Trust No. 1 and Trust No. 2 have different grantors and the same beneficiary. d. Trust No. 1 and Trust No. 2 have different grantors and the same fiduciary and beneficiary.

78.

Inday is a resident citizen of the Philippines. Data for a year: Gross income from business P 700,000 Royalty from books 40,000 Gain on direct sale to buyer of shares of stock of a domestic corporation held as capital asset 70,000 Loss on sale of land in the Philippines held as capital asset with cost of P1,500,000 when the zonal value is P1,200,000 500,000 Business Expenses 300,000 How much is the total income tax expense for the year? a. P177,500 c. P159,500 b. P80,000 d. P156,000 Gross Income from Business Less: Business Expense Personal Exemptions Net Taxable income

700,000 300,000 50,000

Tabular Schedule: 1st 250,000 Tax is Excess of 250,000 - 350,000 = 100,000 x 30% Capital Gains Tax on Shares (70,000 x 5%) Royalty Income (40,000 x 10%) Loss on Sale (Zonal Value 1.2 x 6%) Total Income Tax Expense 79.

350,000 350,000

50,000 30,000 3,500 4,000 72,000 159,500

Mercy is a citizen and resident of the Philippines. She had a compensation income (net of exclusions) of P200,000 and a net income from business of P700,000 for a year. She made quarterly income tax payments amounting to P237,000 and her employer withheld P25,000 on her compensation income. The income tax payable (refundable) for the year is: a. (P25,000) c. (P42,000) b. P237,000 d. (P37,500) Compensation Income Net Income from Business Total Income Basic Exemptions Taxable Income 1st 500,000 Tax 125,000 Excess of 500,000- 850,000 x 32% 112,000 Total Tax Due 237,000 Less: Taxes Payments Quarterly Payments 237,000 Compensation CWT 25,000 262,000 Refundable /Creditable Tax (25,000)

200,000 700,000 900,000 (50,000) 850,000

80.

EMT has the following data on his passive income earned during the year 2016: Philippines Abroad Interest income from bank deposits 45,000 25,000 Interest income from FCDU 50,000 -0Royalties from books 20,000 30,000 Royalties from computer programs 20,000 40,000 Dividend income from a domestic corporation 27,000 13,000 Dividend income from a foreign corporation 33,000 22,000 How much is the resident citizen? a. P21,450 b. P20,400

final

Philippines Interest income from bank deposits Interest income from FCDU Royalties from books Royalties from computer programs Dividend income from a domestic corporation Total Withholding Taxes 81.

withholding

tax

if

the c. d.

45,000 50,000 20,000 20,000 27,000

x x x x

20% 7.5% 10% 20%

x 10%

taxpayer

is

a

P17,700 P36,250

= = = =

9,000 3,750 2,000 4,000

=

2,700 21,450

How much is the final withholding tax if the taxpayer resident alien? a. P21,450 c. P17,700 b. P20,400 d. P36,250 SAME COMPUTATION AS RESIDENT CITIZEN

is

a

82.

How much is the final withholding tax if the taxpayer is a nonresident citizen? c. P17,700 a. P21,450 b. P20,400 d. P36,250 Philippines Interest income from bank deposits 45,000 x 20% = 9,000 Royalties from books 20,000 x 10% = 2,000 Royalties from computer programs 20,000 x 20% = 4,000 Dividend income from a domestic corporation 27,000 x 10% = 2,700 Total Withholding Taxes 17,700 83.

How much is the final withholding tax if the taxpayer is a nonresident alien engaged in trade or business? a. P21,450 c. P17,700 b. P20,400 d. P36,250

Philippines Interest income from bank deposits Royalties from books Royalties from computer programs Dividend income from a domestic corporation Total Withholding Taxes 84.

45,000 20,000 20,000 27,000

86.

x 20%

= = =

9,000 2,000 4,000

=

5,400 20,400

How much is the final withholding tax if the taxpayer is a nonresident alien not engaged in trade or business? a. P21,450 c. P17,700 b. P20,400 d. P36,250

Interest income from bank deposits Royalties from books Royalties from computer programs Dividend income from a domestic corporation Dividend income from a foreign corporation TOTAL PASSIVE INCOME Final Withholding Tax Rate Total Withholding Taxes 85.

x 20% x 10% x 20%

45,000 20,000 20,000 27,000 33,000 145,000 25% 36,250

Nonoy is an employee of a firm in Quezon City. He is supporting his 4 year old brother who is living with him. Data on his compensation income for the year shows: Regular Salary P 240,000 Thirteenth month pay 20,000 Quarterly bonus 40,000 Payroll Deductions: SSS Premiums 3,000 Philhealth contributions 1,200 Pagibig contributions 4,000 Labor union dues 1,000 Premium payments on hospitalization insurance 3,000 Payment of loan 5,000 How much is the taxable income? a. P210,800 c. P208,400 b. P207,800 d. P260,800

Regular Salary Total Income Less: Basic Exemption SSS Premiums Philhealth contributions Pagibig contributions Labor union dues Net Taxable Income

P 240,000 240,000 (50,000) (3,000) (1,200) (4,000) (1,000) 180,800 +30,000 if old laws = 218,000

87.

A domestic corporation, in its fifth (5th) year of operations, had the following data for the year: Net sales P 2,000,000 Capital gain on direct sale to a buyer of shares of a domestic corporation for P500,000

200,000

Capital gain on sale thru a real estate broker of land and building outside the Philippines for P5,000,000

1,000,000

Dividend from a domestic corporation 50,000 Interest on bank deposit 40,000 Cost of sales 600,000 Quarterly corporate income tax paid 190,000 Operating expenses 500,000 The income tax payable upon filing of the annual income tax return is: a. P425,000 c. P570,800 b. P380,000 d. P520,800 Net Sales Cost of Sales Operating Expenses Net Income from Ordinary Business Add Other Income Broker Outside Phils Total Taxable Income Corporate Tax Rate Income Tax Due Less Quarterly Income Tax Paid Income Tax Due and Payable 88.

The Kultura Foundation of the Philippines, a non-stock, non-profit corporation, organized and operated exclusively to preserve and show-case Philippine cultural practices, music, dances, and folk arts, deriving funding from mostly donations, had the following data for the year:

Donations received Interest income from bank deposit Rent income from properties received as donation (net of 5% withholding tax) Expenses related to its rent income 89. a. b.

2,000,000 (600,000) (500,000) 900,000 1,000,000 1,900,000 30% 570,000 (190,000) 380,000

P

20,000,000 100,000 475,000 30,000

How much is the income tax expense of the Corporation for the year? P161,000 c. P172,000 P136,000 d. P125,600

Rent Income (Gross Up) Less: Expenses Net Rent Income

500,000 (30,000) 470,000

Tax Rate Tax Expense Add Interest Income Tax Total Income Tax Expense

30% 141,000 20,000 161,000

90.

A tax imposed in the nature of a penalty to the corporation to deter tax avoidance of shareholders who avoid paying the dividends tax on the earnings distributed to them by the corporation. a. Minimum corporate income tax b. Optional corporate income tax c. Improperly accumulated earnings tax d. Capital gains tax

91.

First statement: An accumulation of earnings or profits (including undistributed earnings or profits of prior years) is unreasonable if it is not necessary for the purpose of the business, considering all the circumstances of the case. Second statement: The term "reasonable needs of the business" are hereby construed to mean the immediate needs of the business, including reasonably anticipated needs. a. Only the first statement is correct b. Only the second statement is correct c. Both statements are correct d. Both statements are incorrect

92.

The Improperly Accumulated Earnings Tax (IAET) is imposed on improperly accumulated taxable income earned starting January 1, 1998 by domestic corporations as defined under the Tax Code and which are classified as closely-held corporations at the rate of: twenty percent (20%). c. ten percent (10%). fifteen percent (15%). d. five percent (5%).

a. b. 93.

The Improperly Accumulated Earnings Tax shall not apply to which of the following corporations? a. Banks and other non-bank financial intermediaries b. Insurance companies c. Publicly-held corporation d. All of the choices

94.

First statement: Once the profits have been subjected to improperly accumulated earnings tax, the same shall no longer be subject to the same tax in later years even if not declared as dividends. Second statement: Profits which have been subjected to improperly accumulated earnings tax when finally declared as dividends shall be subject to tax on dividends. a. Both statements are true b. Both statements are false c. Only the first statement is true d. Only the second statement is true

95.

To avoid payment of IAET, when must the dividends be declared and paid or issued? a. Not later than one year following the close of the taxable year b. Not later than the 15th day following the close of the taxable year c. Not later than 60th day following the close of the taxable quarter d. None of the choices

TAX REMEDIES 96. Rosalie, a compensation income earner, filed her income tax return for the taxable year 2013 on March 30, 2014. On May 20, 2017, Rosalie received an assessment notice and letter of demand covering the taxable year 2013 but the postmark on the envelope shows April 10, 2017. Her return is not a false and fraudulent return. Can Rosalie raise the defense of prescription? a. No. The 3 year prescriptive period started to run on April 15, 2014, hence, it has not yet expired on April 10, 2017. b. Yes. The 3 year prescriptive period started to run on April 15, 2014, hence, it had already expired by May 20, 2017. c. No. The prescriptive period started to run on March 30, 2014, hence, the 3 year period expired on April 10, 2017. d. Yes. Since the 3-year prescriptive period started to run on March 30, 2014, it already expired by May 20, 2017. 97.

On March 30, 2012 Emmett Foods, Inc. received a notice of assessment and a letter of demand on its April 15, 2009 final adjustment return from the BIR. Emmett Foods then filed a request for reinvestigation together with the requisite supporting documents on April 25, 2012. On June 2, 2012, the BIR issued a final assessment reducing the amount of the tax demanded. Emmett Foods was satisfied with the reduction, it did not do anything anymore. On April 15, 2017 the BIR garnished the corporation's bank deposits to answer for the tax liability. Was the BIR action proper? a. Yes. The BIR has 5 years from the filing of the protest within which to collect. b. Yes. The BIR has 5 years from the issuance of the final assessment within which to collect. c. No. The taxpayer did not apply for a compromise. d. No. Without the taxpayer’s prior authority, the BIR action violated the Bank Deposit Secrecy Law.

98.

Renesmee, Inc. received a notice of assessment and a letter from the BIR demanding the payment of P3 million pesos in deficiency income taxes for the taxable year 2015. The financial statements of the company show that it has been suffering financial reverses from the year 2016 up to the present. Its asset position shows that it could pay only P500,000.00 which it offered as a compromise to the BIR. Which among the following may the BIR

require to enable it to enter into a compromise with Renesmee, Inc.? a. Renesmee must show it has faithfully paid taxes before 2016. b. Renesmee must promise to pay its deficiency when financially able. c. Renesmee must waive its right to the secrecy of its bank deposits. d. Renesmee must immediately deposit the P500,000 with the BIR. 99.

100.

As a rule, within what period must a taxpayer elevate to the Court of Tax Appeals a denial of his application for refund of income tax overpayment? a. Within 30 days from receipt of the Commissioner’s denial of his application for refund. b. Within 30 days from receipt of the denial which must not exceed 2 years from payment of income tax. c. Within 2 years from payment of the income taxes sought to be refunded. d. Within 30 days from receipt of the denial or within two years from payment. What is the effect on the tax liability of a taxpayer who does not protest an assessment for deficiency taxes? a.

b. c. d.

101.

The taxpayer may appeal his liability to the CTA since the assessment is a final decision of the Commissioner on the matter. The BIR could already enforce the collection of the taxpayer's liability if it could secure authority from the CTA. The taxpayer's liability becomes fixed and subject to collection as the assessment becomes final and collectible. The taxpayer's liability remains suspended for 180 days from the expiration of the period to protest.

The taxpayer seasonably filed his protest together with all the supporting documents. It is already July 31, 2017, or 180 days from submission of the protest but the BIR Commissioner has not yet decided his protest. Desirous of an early resolution of his protested assessment, the taxpayer should file his appeal to the Court of Tax Appeals not later than a. August 31, 2017. b. August 30, 2017. c. August 15, 2017. d. August 1, 2017.

DOCUMENTARY STAMP TAX 102. A newly formed corporation issued shares of stocks to its incorporators for P150,000. The par value of the shares issued is P100,000. How much is the documentary stamp tax? a. P500 c. P750 b. P1,000 d. P1,500 Par Value Divide by 200 x 1 peso 103.

= 100,000 / 200 = 500

Mr. T invested in shares of stock of Kapisananngmga Sisters Inc. amounting to P100,000 with par value of P80,000. After 2 years, he disposed said shares directly to Mr. B for P230,000. The documentary stamp tax on above transaction is: a. P400 c. P300 b. P1,150 d. P862.50 **80,000 / 200 = 400 x 75% = 300

104.

Based on number 97, but assuming the shares are without par value, how much is the documentary stamp tax? a. P500 c. P750 b. P1,000 d. P0.00 **Selling Price

105.

-

Cost = 230,000 - 80,000 = 150,000 / 200 = 750

Continuing number 99, if the shares were subsequently sold for P200,000, how much is the documentary stamp tax? a. P750 c. P1,500 b. P187.50 d. P375 **Doc Stamp Upon Original Issuance

106.

750 x 25% = 187.50

Mr. Purisima owns a resthouse in Pampanga acquired by him for P10,750,000. He sold the same to Mr. Apo for P5,000,000. The fair market value at the time of sale per assessor’s office is P10,000,000 while zonal value is P15,000,000. The documentary stamp tax on the transaction is: a. P75,000 c. P150,000 b. P225,000 d. P161,250 **15,000,000/1000 = 15,000 x 15 pesos = 225,000

107.

Who is liable to the payment documentary stamp tax? a. Mr. Purisima b. Mr. Apo c. It depends on the agreement of the parties d. It depends on who is benefiting on the transaction

TAXATION QUIZZER PART 2 1.

The BIR is assessing deficiency withholding tax on total Repairs and Maintenance claimed by non-stock non-profit organization. The BIR alleges that foundation failed to withhold the correct amount of the 2% expanded withholding tax due on the Repairs and Maintenance expense. As a CPA assisting the foundation in the BIR tax investigation, you can reason that ___. a. The BIR should have considered the 1% expanded withholding tax instead of the 2%in determining the deficiency tax. b. The foundation is exempt from the tax and hence is not liable to the deficiency withholding tax. c. The foundation is exempt from tax and hence is not required to withhold tax on its expenses. d. The BIR should have considered both the 1% and 2% withholding tax in determining the deficiency tax.

2.

Which of the following statement is true? a. Taxes are in the nature if contracts between the taxpayers and the government. b. Taxes and debts are similar nature and character. c. As the general rule, no set-off is allowed against the demands for taxes levied for general or local government purposes. d. In taxation, the personal consent of the individual taxpayers is required.

3.

Which of the following is not an attribute of tax? a. It is an enforced contribution on a person taxed. It is levied by the executive department of the state. b. c. It is imposed by the state which has jurisdiction person, property, or excises. d. It is generally in money

over

the

4.

Which of the statement below is grammatically correct? a. The imposition of minimum corporate income tax may be suspended if substantial losses are sustained due to a prolonged labor dispute. b. The imposition of minimum corporate income tax maybe suspend if substantial losses are sustained due to prolonged labor dispute. c. The imposition of minimum corporate income tax may be suspended if substantial losses are sustained due to a prolong disputes. d. The imposition of minimum corporate income tax may be suspended. If substantial loss are sustained due to a prolonged labor dispute.

5.

Which of the following statements is correct? a. If what is delegated is tax administration, the delegation is invalid. b. If what is delegated is tax administration, the delegation is valid. c. Tariff powers cannot be delegated to the President. d. As a general rule, taxation can be further delegated.

6.

The BIR issued a tax assessment against the taxpayer who was not given sufficient time to protest the said assessment. The taxpayer noted that their competitors were issued tax assessments but were given enough time to protest. The BIR violated the _____________ of the constitution. a. Equal protection clause c. Equitability principle b. Due process clause d. Uniformity principle Items 7-10 are based on the following information: On March 30, 2012, XXX, Inc., received a notice of assessment and a letter of demand on its April 15, 2009 final adjustment return from the BIR. XXX, Inc., then filed a request for reinvestigation together with the requisite supporting documents on April 25, 2012. On June 2, 2012, the BIR issued a final assessment reducing the amount of the tax demand. Since XXX, Inc., was satisfied with the reduction, it did not do anything anymore. On April 15, 2017 the BIR garnished the corporation’s bank deposits to answer the liability. 7.

Was the BIR action proper? a. No, the taxpayer did not apply for the compromise. b. Yes, the BIR has 5years from the filing within which to collect. c. No, without the taxpayer’s prior authority, the BIR action violated the Bank Deposit Secrecy Law. d. Yes, the BIR has 5 years from the issuance if the final assessment within which to collect.

8.

What is the effect of the XXX, Inc.’s failure to file a protest on its assessed deficiency taxes? a. The taxpayer may file a motion for reconsideration to the CIR on the matter. b. The taxpayer may appeal his liability to the CTA since the assessment is a final decision of the Commissioner on the matter. The taxpayer’s liability becomes fixed and subject to collection c. as the assessment becomes final and collectible. d. The BIR could already enforced the collection of the taxpayer’s liability if it could secure authority from the CTA.

9.

Which statement is correct? The collection of a deficient tax assessment by distraint and levy: a. May be done only once during the taxable year. b. Must be done successively, first by distrait and then by levy; c. May be repeated, if necessary, until the full amount due, including all expenses, is collected; d. Automatically covers the bank deposits of a delinquent taxpayer.

10.

Which of the following is grammatically correct? a. Tax assessment refer to the process of determining the correct amount of tax due in accordance with the prevailing tax laws. b. Tax assessment refer to the process of determining the correct amount of tax due in accordance with the prevailed tax laws.

c. d.

Tax assessment refers to the process of determining the correct amount of tax due in accordance with the prevailed tax laws. Tax assessment refers to the process of determining the correct amount of tax due in accordance with the prevailing tax laws.

11.

A joker was commissioned in a kiddy party to perform magic. The comic was to be paid P100, 000 for his performance and the parties signed the necessary contract. He then gratuitously assigned his rights under the contract to his son. The son later on collected the P100, 000 talent fee of his father from the contractee. The national internal revenue tax/es payable is/are: a. Income tax only. b. Donor’s tax only. Both income and donor’s taxes. c. d. Neither income tax nor donor’s tax.

12.

Which one of the following statements is wrong? a. Income out of the labor of the wife is conjugal property. b. Income out of the separate property of the husband is conjugal property. c. Amount receivable as retirement benefit under R.A. No. 4917 during the marriage is conjugal property. d. Property received is donation when the fair value was P2, 000,000, resulting in a gain of P500, 000. The gain is conjugal property.

13.

Amount receivable by the estate of the deceased, his executor or administration as a beneficiary under life insurance policy taken by the decedent upon his own life is: a. Excluded from gross state. b. Part of gross state if the beneficiary is revocable. c. Excluded from gross state if the beneficiary is irrevocable. Part of gross state whether the beneficiary is revocable or d. irrevocable.

14.

The following journal entry was made in the purchases journal of a VAT-registered taxpayer:

Purchases Cash or Accounts Payable

xxx xxx

The journal entry signifies that: a. Purchases were from a non-VAT supplier. b. Purchases were from a VAT-registered supplier. c. Purchases were either from non-VAT or VAT-registered supplier. d. Input tax has been taken as part of the cost of purchase. 15.

A common carrier by land is engaged in the transport of passengers, goods and cargoes. He is not VAT-registered. What business tax or taxes is he liable to pay? a. 12% value-added tax b. 3% common carrier’s tax

c. d.

12% VAT on gross receipts from transport of goods and cargoes and 3% common carrier’s tax on gross receipts from transport of passengers. 3% tax on VAT-exempt persons on gross receipts from transport of passengers.

16.

Statement 1. As to the property of the state, exemption is the rule and taxation the exception. Statement 2. As of the property of the taxpayer, taxation is the rule and exemption the exception. a. b. c. d. 17.

Both statements are correct Both statements are incorrect. Statement 1 is correct while statement 2 is incorrect. Statement 1 is incorrect while statement 2 is correct.

Foreign income taxes paid by the resident citizen or domestic corporation. a. May be claimed only as tax credit. b. May be claimed only as tax deduction. c. Do not qualify either as a tax credit or as a tax deduction. May be claimed either as a tax credit or as a tax deduction at d. the option of the income taxpayer.

18.

Statement 1. Government agencies performing essential governmental functions are subject to tax unless expressly exempted. Statement 2. Government agencies performing propriety function are exempt from tax unless expressly taxed. a. b. c. d.

Both statements are Both statements are Statement 1 is true Statement 2 is true

correct. incorrect. while statement 2 is false. while statement 1 is false.

19.

Statement 1. Tax avoidance or tax minimization is the use by the taxpayer of legally permissible methods in order to reduce tax liability. Statement 2. Tax evasion or tax dodging is the use by the taxpayer of illegal means to defeat or lessen the payment of tax. a. b. c. d. 20.

Both statements are correct. Both statements are incorrect. Statement 1 is correct while statement 2 is incorrect. Statement 2 is correct while statement 1 is incorrect.

Which one among the following items below is included in the gross state? a. Revocable transfer. b. Transfer with reservation of certain rights. c. Transfer under general power of appointment. d. Transfer in contemplation of death which is onerous.

21.

Which of the following statement is correct? a. The final tax on compensation of special kind of non-resident aliens is 25% of the gross income. b. Interest income from a foreign currency deposit unit in the Philippines of a non resident alien is not subject to final tax. c. Informer’s reward is subject to final tax of 10% based on the 10% of the value of tax assessed or P1, 000,000 whichever is higher. d. Prizes exceeding P10,000 derived by non-resident alien not engage in trade or business here in the Philippines is subject to a final tax of 20%

22.

The power to decide disputed assignment, refunds of internal revenue taxes, fees or other charges, penalties imposed in relation thereof, or other matters arising other the tax code or other laws or portions thereof administered by the BIR is vested in the: a. Secretary of Finance b. Commissioner of Internal Revenue c. Court of Tax Appeals d. Regional Trial Court

23.

The commissioner of the Internal Revenue (CIR) is prohibited by law to look into the bank accounts of taxpayer, except when: a. Taxpayer is accused of heinous crime b. Taxpayer did not invoke his right to privacy during the tax audit. c. Taxpayer applies for compromise on tax obligation on account of financial incapacity. d. The CIR has reason to believe that taxpayer has filed a false or fraudulent return.

24.

A VAT-registered realty company sells real property in the course of its business. On April 30, 2016, it has sold a lot under the following items (VAT excluded). Selling Price Down payment, 4/30/16 1st Installment, 4/30/17 2nd Installment, 4/30/18 3rd Installment, 4/30/19 4th Installment, 4/30/20 Interest and other charges of 10% On unpaid balance per installment

P4,000,000 400,000 600,000 1,000,000 1,000,000 1,000,000

The zonal value of the lot at the time of sale is P4, 800,000. The output VAT for the installment received on April 30, 2017, is: a. P43,200 b. P86,400 c. P115,200 d. P129,600

Output Tax (4,800,000 x 12%) = 576,000 Principal 600,000 / 4,000,000 x 576,000 Interest 3,600, 000 x 10% x 12% Total 25.

86,400 43,200 129,600

Counting No. 24, the output VAT on April 30, 2020, is: a. P132, 000 b. P144,000 P156,000 c. d. P228,000

4,800,000 x 12% = 576,000 out put tax Principal 1,000,000 / 4,000,000 x 576,000 Interest 1,000,000 x 10% x 12% Total 26.

a. b. c. d.

= =

144,000 12,000 156,000

Assuming that the real property in No. 24 is the residential lot sold for cash of P1, 750,000 (VAT not separately blend in the sales document) on April 30, 2016. Zonal value of the lot at the time of sale is P1, 900,000. The output VAT on the sale is: P187,500 P210, 000 P228,000 None, as it is exempt the VAT.

27.

a. b. c. d. 28.

= =

The lease of real or personal property is subject to: i. 7% gross receipt tax (GRT) if the lessor is the bank. ii. 3% percentage tax or 12% value-added tax (VAT) if lessor is not a bank.

the

No to I and II Yes to I and II Yes to I only Yes to II only

Ayala Land, Inc. (ALI) bought a parcel of land in 2014 for P7 million as part of its inventory of real properties. In 2016, it sold the land for P12 million which was its zonal valuation. In the same year, it suffered a loss of P6 million for selling another parcel of land from its inventory. These were the only transactions ALI had in its real estate business. Which of the following is the applicable tax treatment? a. ALI shall be subject to a tax of 6% of P12 million. b. ALI’s gain of P5 million shall be subject to holding period. c. ALI could deduct its P6 million loss from its P5 million gain. d. ALI’s P6 million loss could not be deducted from its P5 million gain. 29. This is an inherent limitation on the power of taxation: a. The rule of taxation shall be uniform and equitable. b. No law impairing the obligations of contacts shall be enacted.

c. Tax laws can not apply to the property of foreign governments. d. Charitable institutions, churches, parsonages, convents and all lands, buildings and improvements, actually, directly, and exclusively used for religious, charitable or educational purposes shall be exempt for taxation. 30.

a. b. c. d. 31.

a. b. c. d. 32.

X took a life insurance policy of P5 million where the monthly premium is P10, 000. The proceeds will be paid to X after 25 years to the X’s estate should X die before completing the equivalent of 25 years payment. If the X outlived the policy, which of the following is correct? The proceeds will be part of X’s gross estate. The proceeds will be part of X’s taxable income. The proceeds will be party taxable estate and partly exempt. The proceeds will be party taxable income and partly exempt. Containing the preceding number, except that after paying the equivalent of ten years premium, X transferred the policy to Y for P1.5 million and Y continued paying the monthly premium as they mature. After 10 years, X died. Which of the following is correct? The proceeds will be part of X’s gross estate. The proceeds received by Y is part of his taxable income. The amount received by X from Y is part of X’s taxable income. The amount received by the X from Y and the proceeds received by Y are partly taxable income and partly exempt. An owner of several warehouses for rent, which used to be VATexempt because its annual gross receipts never exceeded P1,919,500 decided to register under the VAT system on January 2, 2016. The following data were from the first quarter ending March 31, 2016: Rental from warehousing services, net of VAT Purchases of supplies, gross of VAT Inventory of supplies, January 1, 2016 Actual VAT paid on the inventory of supplies, January 1, 2016

P 672,000 224,000 201,600 21,600

The value added tax payable for the quarter is: a. P23,520 b. P35,040 c. P41,088 d. P52,608 VAT on rental (672,000 x 12 %) VAT on purchase of supplies (224,000 x 12/112) Transitional Input Tax(higher) VAT Payable

80,640 (24,000) (21,600) 35,040

33.

Assuming that one vacant warehouse in the preceding number was rented out for the whole month of April 2016 and received P107, 000 as rent, gross of VAT but net of the applicable creditable withholding income tax (expanded). The output VAT on the rental receipt is: a. P12,000 b. P12,240 c. P12,840 d. P13,440 107,000/107% = 100,000 x 12% = 12,000

Numbers 34 to 42 are based on the following information: Dina Cabangon, a citizen and resident of the Philippines, died on November 1, 2016. Her marriage was under the system of absolute community of property. The following properties and obligations were left: Property received by Dina as inheritance on February 1, 2015 (during the marriage) Real property acquired through the labor of both Dina and her Husband during the marriage (family home) Property owned by Dina before marriage Property owned by Dina’s Husband before marriage Funeral expenses Unpaid mortgage on property inherited Judicial expenses for the settlement of the estate Unpaid obligations (excluding the unpaid mortgage)

P2,000,000 4,000,000 300,000 200,000 300,000 200,000 80,000 40,000

The property received as inheritance was part of the gross estate of the prior decedent at a fair market value of P1,100,000. At the time of inheritance, it was mortgage for P300,000. Dina was able to pay P100,000 before she passed away. 34.

The total community property is: a. P6,500,000 c. P4,000,000 b. P4,500,000 d. P2,300,000

35.

The total exclusive property is: a. P4,200,000 c. P2,300,000 b. P2,500,000 d. P2,000,000

36.

The total ordinary community deduction is: a. P200,000 c. P420,000 b. P320,000 d. P445,000

37.

The total ordinary deduction) is: a. P400,000 b. P300,000

exclusive c. P200,000 d. P100,000

deduction

(excluding

vanishing

38.

The deduction for family home is: a. P0 c. P2,000,000 b. P1,000,000 d. P4,000,000

39.

The amount of vanishing deduction is: c. P736,000 a. P0 b. P816,000 d. P656,000

40.

The total special deduction is: a. P0 c.P2,000,000 b. P1,000,000 d.P4,000,000

41.

The taxable net estate is: a. P1,154,000 c. P3,154,000 b. P2,154,000 d. P4,244,000

SOLUTION: Separate 2,000,000 300,000 200,000 2,000,000

Gross Estate Funeral Expenses: Actual 300,000 Limit (6.5 x 5%) 325,000 Threshold 200,000 Whichever is lower Judicial Expenses Indebtedness (200,000) Vanishing Deductions* (736,000) Net Estate After Ordinary Deductions 1,064,000 Family Home Standard Deductions Share of Surviving Spouse 1/2 x 4,180,000 Taxable NE *Vanishing Deduction: Lower FMV Mortgage Assumed and Paid Initial Basis Pro Rated Deductions: 1,000,000/6,500,000 x 520,000 Final Basis Vanishing Rate Vanishing Deductions 42.

Common 4,000,000

Total

4,500,000

6,500,000

(200,000) (80,000) (40,000)

4,180,000

5,244,000 (1,000,000) (1,000,000) (2,090,000) 1,154,000

1,100,000 (100,000) 1,000,000 (80,000) 920,000 _ 80%__ 736,000

Going back to the original problem, except that the marriage of Dina Cabangon to her spouse was under the system of conjugal partnership of gains. The taxable net estate would be: a. P3,206,032 c. P1,206,032 b. P2,206,033 d. P1,154,000

Separate 2,000,000 300,000 2,300,000

Gross Estate Funeral Expenses: Actual 300,000 Limit (6.5M x 5%) 325,000 Threshold 200,000 Whichever is lower Judicial Expenses Indebtedness (200,000) Vanishing Deductions** (733,968) Net Estate After Ordinary Deductions 1,366,032 Family Home Standard Deductions Share of Surviving Spouse 1/2 x 4,180,000 Taxable NE **Vanishing Deduction: Lower FMV Mortgage Assumed and Paid Initial Basis Pro Rated Deductions: 1,000,000/6,300,000 x 520,000 Final Basis Vanishing Rate Vanishing Deductions 43.

Common 4,000,000

Total

4,000,000

6,300,000

(200,000) (80,000) (40,000) 3,680,000

5,046,032 (1,000,000) (1,000,000) (1,840,000) 1,206,302

1,100,000 (100,000) 1,000,000 (82,540) 917,460 80% 733,968

In March 2016, Imelda, who is fond of jewelries, bough the following: diamond ring for P750, 000; bracelet for P250, 000; necklace for P500, 000; and a brooch for P500, 000. Imelda drives income from the exercise of her profession as a topnotch Interior Designer. In October 2016. Imelda sold her diamond ring, bracelet and necklace for only P1.25 million, incurring a loss of P250, 000. She used the P1.25 million to buy a solo diamond ring in November 2016 which she sold for P1.5 million in September 2017. Imelda had no other transaction on jewelry in 2017. Which among the following best describes the tax implications arising from the aforesaid transactions?

a. Imelda may carry over and deduct her 2016 loss only from her 2017 gain. b. Imelda may deduct her 2016 loss from both her 2017 professional income. c. Imelda may not deduct her 2016 loss from both her 2017 professional income and her gain. d. Imelda may carry over and deduct her 2016 loss from her 2017 professional income as well as from her gain.

44.

Taxation could be exercise by the following except one. Which one? a. Judiciary b. Legislative c. Local government unit d. President of the Philippines, in certain cases.

45.

Stages, aspects or processes in taxation. a. Levy of the tax b. Collection of the tax. c. Payment of the tax by the taxpayer. d. All of the above.

46.

Statement 1. Onerous donations are subject to donor’s tax. Statement 2. Gratuitous donations are not subject to donor’s tax. a. Both statement s are true b. Both statements are false c. Only statement 1 is true but not statement 2. d. Only statement 2 is true but not statement 1.

47.

Which of the following statements are correct? a. Gift-splitting is a form of tax dodging. b. The uncle who is the brother of the donor’s mother-in-law is a non-stranger to the donor for purposes of the donor’s tax. c. A gift made to a relative in January 2016 is to be added to the gift made to the same relative in December 2015 in determining the gift tax. d. Renunciation by an heir including the surviving spouse of his/her share in the hereditary estate left by the decedent is subject to donor’s tax if done in favor of identified heirs to the exclusion or disadvantage of the other co-heir/s in the hereditary estate.

48.

Mistah, single and sales executive of a leading pharmaceutical firm (RiteMed), received in 2016 the following from his employer:

Salary, net of P267,000 withholding tax Allowances and benefits received:  Rent paid by RiteMed on the house which Mistah occupies for residential purposes, net of 5% withholding

P 683,000

129,200

 Entertainment allowance subject to liquidation (P75,000 was duly receipted in the name of RiteMed and used to entertain RiteMed’s customers and the balance of P25,000 was used to purchase a late model mobile phone for the personal use of Mistah) 100,000  Reimbursement of entertainment expenses paid by Mistah (P17,500 was used to entertain Mistah’s boyhood pals and the balance of P22,500 was used to promote RiteMed’s businesses.) 40,000

 Fixed yearly allowance for entertainment

85,000

The fringe benefit tax is: a. P50,400 b. P52,000 c. P84,000 d. P92,000 Gross Rent on Housing 129,200/95% = 136,000 x 50% = Expense Account (CP & Pal Exp) 25,000 + 17500 Total 110,500/68% 162,500 x 32% 49.

68,000 42,500 110,500

162,500 52,000

Continuing number 48, the income tax payable by Mistah is: a. P13,200 old law b. P27,760 c. P29,200 d. P43,360 Compensation(683,000 + 267,000) Fixed Year Allowance Personal Exemptions Taxable Income

950,000 85,000 (50,000) 985,000

Income Tax per Tabular (OLD LAW) up to 500,000 Excess (485,000 x 32%) Withholding Tax Income Tax Payable

125,000 155,200 (267,000) 13,200

Income Tax per Tabular (TRAIN LAW) up to 800,000 Excess (185,000 x 30%) Withholding Tax Income Tax Payable (Refund)

130,000 55,500 (267,000) (81,500)

Numbers 50 to 56 are based on the following information: Domestic Sales to private entities P500,000 Sales to the government 500,000 Sales of exempt goods 500,000 Input taxes passed on by VAT-registered suppliers on: Sales to private entities 30,000 Sales to the government 25,000 Sales of exempt goods 10,000 Purchase of depreciable capital goods not attributable to any specific activity (monthly amortization for 60 months) 120,000

Export P500,000

20,000

The sales to the government were subjected to the automatic deduction of the 1% creditable withholding tax (CWT) on its purchases from domestic suppliers. 50. a. b. c. d.

The value-added entities is: Zero (P10,000) P60,000 P70,000

tax

payable

on

the

domestic

sales

to

private

51. a. b. c. d.

The total input taxes attributable to zero-rated sales is: P20,000 P30,000 P50, 000 P60, 000

52.

If the input taxes attributable to zero-rated are claimed as tax credit, the net input value-added tax refundable is: Zero P40,000 P50,000 P60,000

a. b. c. d. 53. a. b. c. d. 54. a. b. c. d. 55.

The actual input taxes attributable to the domestic sales to the government is: P25,000 P30,000 P55,000 P65,000 The value-added tax payable on the domestic sales to the government which was withheld as final withholding VAT is: P5,000 P25,000 P30,000 P35,000

The journal entry to take up the domestic sales to the government is: a. Cash/AR 560,000 Sales 560,000 b. Cash/AR 560,000 Sales 500,000 Output tax 60,000 c. Cash/AR 535,000 Final withholding VAT 25,000 Sales 500,000 Output tax 60,000

d. Cash/AR Final withholding VAT Creditable withholding tax Sales Output tax

OT 500,000 x 12% 500,000 x 0% IT 500,000 x 7% 120,000 x 1 / 4 VAT Payable Excess Input Tax FINAL VAT Withheld 56. a.

b.

c. d.

530,000 25,000 5,000 500,000 60,000

Private Entities VAT 0% VAT 60,000 -0(30,000) (20,000)

Government 60,000 (35,000)

(30,000) -0-

(30,000) (50,000) 25,000

The journal entry to reflect the excess of actual input VAT-over the statutorily allowed input tax on the domestic sales to the government is: Output tax 60,000 Revenue and expense summary 20,000 Input tax 55,000 Final withholding VAT 25,000 Output tax 60,000 Input tax 15,000 Final withholding VAT 25,000 Revenue and expense summary 20,000 Output tax 60,000 Input tax 55,000 Revenue and expense summary 5,000 Output tax 60,000 Revenue and expense summary 20,000 Input tax 55,000 Cash/AP 25,000

57. Which one among the following statements is wrong? a. Private Banks may be authorized to collect internal revenue taxes. b. The Bureau of Customs (BOC) is also charged with the collection of internal revenue taxes. c. The local government unit’s (LGUs), such as cities, Municipalities and Provinces, from part of the national tax system. d. The bureau of Internal Revenue (BIR) is part of the administrative machinery for the assessment and collection of internal revenue taxes. 58.

The prescriptive period for the issuance of a formal letter of demand and final assessment notice (FLD/FAN) may not ordinarily be stayed because of the lifeblood theory. There are certain instances, however, where the running of the prescriptive period

a. b. c. d.

may be suspended. Which among the following instances is not among the recognized exceptions which suspend the prescriptive period within which to assess? If the taxpayer is out of the country. If the taxpayer changes his address, informing the commissioner of such change. Where the taxpayer request for and is granted a re investigation by the Commissioner. When the Commissioner of Internal Revenue (CIR) is prevented from making an assessment and within 60 days thereafter.

59.

An examination of a calendar year corporate taxpayer’s records shows that it filed its final adjustment income tax return on February 15, 2016 for its 2015 income. It subsequently filed an amended income tax return March 21, 2016. Up to what date is the Bureau of Internal Revenue (BIR) within which to issue a formal letter of demand and final assessment notice (FLD/FAN)? c. March 21, 2019 a. December 31, 2018 b. February 15, 2019 d. April 15, 2019

60.

A closely held corporation has initially offered its shares in the Philippine stock exchange (PSE). The following data pertain to the initial public offering (IPO): Number of shares sold in IPO Total outstanding shares before the listing in the PSE Gross value in money of the IPO The percentage tax due is: a. P100,000 c. P400,000 d. P800,000 b. P200,000 Gross Value of IPO OPT Rate 1M / 4M = 25% ( 4%, from 4%,2%, 1%) Percentage Tax

61. a. b. c. d. 62.

1,000,000 shares 3,000,000 shares P20,000,000

20,000,000 4% 800,000

Shares of stock in a domestic corporation held as investment when sold not through the local stock exchange shall be subject to: 3% OPT or 12% VAT based on gross income ½ of 1% based on gross selling price or gross value in money. 5% on first P100, 000 capital gain; 10% on excess of P100, 000. 4%;2%;1% based on gross selling price or gross value in money. Malakas and Maganda were legally separated. They have six minor children, all qualified to be claimed as additional exemptions for income tax purposes. The court awarded custody of two of the children to Malakas and three to Maganda, with Malakas directed provide full financial support for them as well. The court awarded the sixth child to Malakas’ parents with Malakas also providing full financial support. Assuming that only Malakas is gainfully earning while Maganda is not, for how many children could

Malakas claim as additional exemptions when he files his annual income tax return? a. Two children c. Five Children b. Three children d. Six children 63.

Statement 1. Health and/or hospitalization insurance premium is deductible from gross income by the spouse who claimed the additional exemptions in case of married income taxpayers. Statement 2. Health and/or hospitalization insurance premium paid by an individual income taxpayer is deductible from gross income for a minimum amount of P2, 400 provided the family’s gross income for the year does not exceed P250, 000. a. Both statements are true b. Both statements are false c. Statement 1 is true while statement 2 is false. d. Statement 2 is true while statement 1 is false.

64.

The following fringe benefits were given by an employer to its employees for the quarter ending September 30, 2016:

De minimis benefits (not exceeding the maximum) Reimbursed expense of rank and file employees Housing benefits to managers and supervisors (Representing total rents)

P 200,000 400,000 680,000

The fringe benefit tax payable for the quarter is: a. P160, 000 c. P442, 353 b. P320, 000 d. P502, 353 Housing Benefits to Manager 680,000 x 50% = 340,000 / 68% x 32% = 160,000 65.

Continuing number 64, how much would be the total deductions from gross income which may be claimed by the employer? a. P1,100,000 c. P1,440,000 b. P1,280,000 d. P1,600,000

De minimis benefits (not exceeding the maximum) Reimbursed expense of rank and file employees Housing benefits to managers and supervisors (Representing total rents) Fringe Benefit Tax Expense Total Deductions from Gross Income 66. a. b. c. d.

P 200,000 400,000 680,000 160,000 1,440,000

Going back to the number 64, the employer shall file a remittance return of the final tax on fringe benefit and pay the tax withheld within: 5 days from the close of each month. 10 days from the close of each month. 10 days from the close of each fiscal quarter. 10 days from the close of each calendar quarter.

67.

Kalansay, a native of Negros, died leaving a property acquired by purchase from Naty Gok who died 3 ½ years ago. The property is now a Kalansay’s gross estate. The estate’s vanishing deduction rate is: a. 0% b. 20% c. 40% d. 60% Property is not gratuitously acquired by the present Estate

68.

A lessor of residential units has the following gross receipts for 2016: Monthly Number Rent per unit of units Total P 9,000 5 P 540,000 10,000 5 600,000 11,000 5 660,000 12,000 5 720,000 The output VAT is: a. P0 c. P270,000 b. P230,340 d. P302,400 Leasing on a residential units is VAT Exempt

69.

Using the same facts in number 68, only that the lessor is a VATregistered person, the OPT tax due is: a. P0 c. P67,500 b. P57,585 d. P75,600 OPT Exempt also

70.

The deduction allowed for the payment of premium on hospitalization insurance during the taxable year by a resident citizen amounting to P3, 000 for the months of August to December is: a. P 3,000 b. P2, 400 c. P1, 200 d. P1, 000 200 x 5 months = 1,000

71.

The following are among the constitutional limitation on the power of taxation. Which one is not? Due process clause Non-impairment clause Equal protection clause No imprisonment for non-payment of tax

a. b. c. d.

72.

a. b. c. d. 73.

Pacmom filed her income tax from return for 2015 on May 16, 2016 and paid the tax of P50,000. Upon audit by the BIR an assessment notice was issued on April 30, 2019, requiring Pacmom to pay a deficiency tax of P 75,000 not later than July 30, 2019. Pacmom will: False prescription as defense Request for an extension of time to pay the deficiency income tax. Go to the court of tax appeals to appeal the assessment made by the BIR. Ignore the assessment as the date of collection is already way beyond three years, covering taxable year 2015. Date assessment was received Date petition for reinvestigation was filed by the BIR Date of filing of documents to support the petition Date decision of denial of the petition was received The last day to appeal to the CTA is on: a. April 17, 2016 c. May 8, 2016 b. April 17, 2016 d. May 28, 2016

March 08, 2016 March 18, 2016 April 08, 2016 April 28, 2016

74.

Using the same facts in number 73, only that instead of going to the CTA, a request for reconsideration was filed with the BIR on May 7, 2016. Date decision of denial of the request for reconsideration was received on June 2, 2016. The last day to appeal to the CTA is on: c. June 23, 2016 a. June 7, 2016 b. June 21, 2016 d. July 2, 2016

75.

Continuing number 74 only that instead of receiving a decision denying a request for reconsideration, a revised assessment was received on June 2, 2016. The last day to appeal to the CTA is on: a. June 7, 2016 c. June 23, 2016 d. July 2, 2016 b. June 21, 2016 30 days from June 2, 2016

76.

Statement 1. The Bureau of Internal Revenue (BIR) has as its powers and duties the assessment and collection of the national internal revenue taxes, fees, and changes and the enforcement of all fortitudes, penalties, and fines connected therewith.

a. b. c. d.

Statement 2. The Bureau of Customs (BOC) has its powers and duties the assessment and collection of awful revenues from imported and exported articles and all other dues, fees, charges, fines, and penalties accruing under the tariff and customs laws. Both statements are correct. Both statements are wrong. Statement 1 is correct but statement 2 is wrong. Statement 2 is correct but statement 1 is wrong.

77. a. b. c. d. 78.

a. b. c. d.

A customs duty that imposes both advance duties on imported or exported articles is: Anti-dumping duty. Countervailing duty. Discriminating duty. Compound customs duty.

and

specific

customs

Consider the following statements: i. The imposition of customs duties also assists in economic development. ii. Customs duties are sometimes imposed to protect local consumers. iii. The purpose of regular customs duties is to raise revenues to meet the needs of government. iv. Compound customs duties are computed only on the basis of units of measure, such as weight, measurement, quantity, etc. All the above statement is correct. Only statements I, II and III are correct. Only statements I, II and IV are correct. Only statement I, III and IV are correct.

Numbers 79 to 80 are based on the following information pertaining to taxable year 2016: Phil. USA Sales P500,000 P600,000 Sales discounts, Returns and allowances 20,000 50,000 Cost of sales 230,000 250,000 Rent expenses 50,000 80,000 Salaries and wages 50,000 60,000 Interest expenses 10,000 Entertainment and presentation 10,000 Contribution to charitable institution 10,000 Interest on bank deposit 10,000 20,000 Royalty- musical compositions 20,000 20,000 Dividends from domestic corporation 10,000 79.

a. b. c. d.

The taxable income if the taxpayer is a domestic corporation (DC) and resident citizen (RC), under:

Itemized Deductions DC RC P320,000 P270,000 P328,150 P278,150 P280,000 P230,000 P288,150 P238,150

ITEMIZED DEDUCTIONS Passive Income Sales

Optional Deduction DC RC P330,000 P280,000 P354,000 P592,000 P378,000 P328,000 P330,000 P568,000 DC 40,000 1,100,000

RC 40,000 1,100,000

Sales Disc, Ret & Allowances Net Sales Cost of Good Sold Business Gross Income

(70,000) 1,030,000 (480,000) 550,000

Total Gross Income Other Expenses Interest 10,000 - (10,000 x 33%) Entert & Repre (1,030,000 x .5%) Net Income Before CC

590,000 (240,000) (6,700) (5,150) 338,150

590,000 (240,000) (6,700) (5,150) 338,150

CC Actual < Limit Personal Exemption Taxable Income

(10,000) -0328,000

(10,000) (50,000) 278,150

OPTIONAL DEDUCTION Passive Income Net Sales Business Gross Income Total Gross Income/ Gross Receipt & Net Sales 40% OSD Personal Exemption Taxable Income 80. a. b. c. d.

DC 40,000

(70,000)____ 1,030,000 (480,000) 550,000

RC 40,000 1,030,000

550,000 590,000 (236,000) -0354,000

1,070,000 (428,000) (50,000) 592,000

The taxable income if the taxpayer is a resident corporation (RFC) and non-resident citizen (NRC): RFC NRC P160,000 P270,000 P130,900 P 80,900 P133,855 P 80,900 P120,000 P 70,000

ITEMIZED DEDUCTIONS Passive Income Sales

500,000

500,000

Sales Disc, Ret & Allowances Net Sales Cost of Good Sold Other Expenses Interest 10,000 - (10,000 x 33%) Entert & Repre (1,030,000 x .5%) Net Income Before CC

(20,000) 480,000 (230,000) (100,000) (6,700) (2,400) 140,900

(20,000) _ 480,000 (230,000) (100,000) (6,700) (2,400) 140,900

CC 140,900 x 5% Personal Exemption Taxable Income

RFC

(7,045) -0133,855

NRC

(10,000) (50,000) 80,900

foreign

TAXATION QUIZZER PART 3 1. The following are common to the inherent power of taxation, power of eminent domain and police power, except for which of the following? a. They are necessary attributes to the sovereignty. b. They interfere with private rights and property. c. They affect all persons or the public. d. They are legislative in implementation. 2. In case of ambiguity, tax laws shall be interpreted: a. Strictly against the taxpayer. b. Liberally against the government. c. Liberally in favor of the taxpayer. d. Liberally in favor of the government. 3. Which of the following statements is not correct? a. Taxes may be imposed to raise revenue or to provide incentives or disincentives for certain activities within the state. b. The state can have the power of taxation even if the constitution does not expressly give it the power to tax. c. For the exercise of the power of taxation, the state can tax anything at anytime. d. The provisions of taxation in the Philippine Constitution are grants of power and not limitation on the taxing power. 4. Which of the following is not a direct tax? a. Immigration tax c. Income tax d. Contractor’s tax (Now VAT) b. Transfer tax 5. In which situation will a CPA’s signature be necessary? a. When the value of the gross estate is P2,000,000 and above. b. When the value of the gross estate exceeds P2,000,000. c. Regardless of the value where the gross estate consists or registered or registrable property. d. When the value of the gross estate exceeds P200,000, although exempt from transfer tax. 6. A seller of goods is non-VAT registered. His annual gross sales amount to P1,919,500. To what business tax is he liable? a. 3% tax on VAT-exempt persons c. 3% common carrier’s tax b. 12% value-added tax d. Not subject to any percentage tax 7. Abigail sold through the local stock exchange, 10,000 PLDT shares that she bought 2 years ago. Abigail sold the shares for P2 million and realized a net gain of P200,000. The transactions is a. Subject to regular income tax rates for individuals but only 50% shall be recognized because it is a long-term capital gain. b. Subject to capital gains tax amounting to P15,000. c. Subject to percentage tax amounting to P10,000. d. Subject to regular income tax rates for individual under Section 24 (A).

8. Proceeds of life insurance to the extent of the amount receivable by the estate of the deceased, his executor or administrator under policies taken out by the dependent upon his own life shall be I. Part of the gross estate irrespective of whether or not the insured retained the power of revocation. II. Not part of the gross estate if the beneficiary is irrevocable. III. Part of the gross income if the designation of the beneficiary is revocable. IV. Not part of the gross income irrespective of whether or not the insured retained the power of revocation. a. I and II b. I and III c. I and IV d. only I 9. One of the following is not correct Deduction Maximum a. Funeral Expenses P200,000 b. Family home P2,000,000 c. Medical expenses P500,000 d. Standard Deduction P1,000,000 10. One of them is not considered non-resident citizen? a. A citizen of the Philippines who establishes to the satisfaction of the Commissioner the fact of his physical presence abroad with a definite intention to reside therein. b. A citizen of the Philippines who leaves the Philippines during the taxable year to reside abroad, either as an immigrant or for employment on permanent basis. c. A citizen of the Philippines who works and derives income from abroad and whose employment thereat requires him to be physically present abroad most of the time during the taxable year. d. A citizen of the Philippines who went on a business trip abroad and stayed therein most of the time during the year. 11. Z is a Filipino immigrant living in the United States for more than 10 years. He is retired and he came back to the Philippines as a balikbayan. Every time he comes to the Philippines, he stays here for about a month. He regularly receives a pension from his former employer in the United States, amounting to USD1,000 a month. While in the Philippines, with his pension pay from his former employer, he purchased 3 condominium units in Makati, which he is renting out for P15,000 a month each. Does the USD1,000 pension become taxable because he is now in the Philippines? a. Yes. Income received in the Philippines by non-resident citizens is taxable. b. Yes. Income received in the Philippines or abroad by nonresident citizens is taxable.

c. No. Income earned abroad by non-resident citizens is no longer taxable in the Philippines. d. No, the pension is exempt from taxation being one of the exclusions from gross income. 12. Which of the following income earners is required to file income tax return? a. Minimum wage earners b. Non-resident alien not engaged in business c. An individual with respect to pure compensation income deriving from such sources within the Philippines, the income tax on which has been correctly withheld and that an individual deriving compensation from one employer at any time during the taxable year d. General professional partnership 13. Which of the following fringe benefit is not subject to fringe benefit tax? a. a. Housing benefit c. De minimis benefit b. b. Expense account d. Vehicle benefit 14. In the case of a taxpayer, only the following percentages of the gain or loss recognized upon the sale or exchange of a capital asset shall be taken into account in computing net capital gain, net capital loss and net income. I. Statement 1 One hundred percent (100%) if the capital asset has been held for not more than twelve (12) months by a taxpayer. II. Statement 2 Fifty percent (50%) if the capital asset has been held for more than twelve (12) months by a taxpayer. c. False, false a. True, true b. True, false d. False, true 15. 15.A, worked for a manufacturing firm but due to business reverses, the firm offered a voluntary redundancy program in order to reduce overhead expenses. Under the program, an employee who offered to resign would be given separation pay equivalent to his 3 months basic salary for every year of service. A accepted the offer and received P800,000 as separation pay under the program. After all employees who accepted the offer were paid, the firm found its overhead still excessive. Hence, it adopted another program, where various unprofitable departments were closed. As a result, B was separated from the service. B also received P800,000 as separation pay. At the time of separation both A and B have rendered at least 10 years of service but A was 55 years old while B was only 45 years old. As a result, a. Both amounts are exempt from income tax b. Both amounts are subject to income tax c. Only Mr. A is subject to income tax d. Only Mr. B is subject to income tax

16. The following shall not be subject to estate tax a. The merger of usufruct in the owner of naked title b. The transmission or delivery of inheritance or legacy by the fiduciary heir or legatee to fideicommissary c. The transmission from the first heir, legatee, or donee in favor of another beneficiary in accordance with the desire of the predecessor d. All bequest, devises, legacies or transfers in social welfare, cultural and charitable institutions 17. Which of the following transactions is subject to zero-rated valueadded tax? a. Services rendered to persons engaged in international shipping or air transport operations b. Services rendered banks, non-bank financial intermediaries c. Generation, transmission and distribution of electricity d. Services rendered by professionals such as CPA, Physicians, and Lawyers. 18. Which of the following businesses is allowed presumptive input valueadded tax? a. Manufacturer of canned goods b. Manufacturer of packed juices c. Manufacturer of packed noodles d. Manufacturer of dried fish 19. Which of the following shall be allowed as a deduction from the gross income? a. Depreciation for vehicle for land transport used for personal purposes the value of which does not exceed P2,400,000. b. Depreciation for vehicle for land transport used in the business the value of which exceeds P2,400,000. c. Depreciation for land vehicles the value of which exceeds P2,400,000 where the taxpayer’s main line of business is transport operations or lease of transportation equipment and the vehicles purchased are used in said operations. d. Depreciation for airplanes and/or aircraft the value of which exceeds P2,400,000 where the taxpayer’s main line of business is sale of goods or properties and the vehicles purchased are used in said operations. 20. A, at the time of retirement, had 1,000 pieces of merchandise which was deemed sold at a value of P20,000, with an output tax of P2,400. After retirement, A sold to B 500 pieces of these for P12,000. In the contract of sale or invoice, A stated sales invoice number wherein the output tax on deemed sale was imposed and the corresponding tax paid on the 500 pieces. He prepared the following invoice: I. Gross selling price P10,800 II. VAT previously paid on deemed sale 1,200 III. Total P12,000

How much is the input tax of B? a. P2,400 b. P1,200 c. P1,000 d. None of the above 21. If a VAT-registered person issues a VAT invoice or VAT official receipt for VAT-exempt transaction, but fails to display prominently on the invoice or receipt the words “VAT-exempt sale”, the transaction shall: a. Still be exempt from value-added tax b. Become taxable and the issuer shall be liable to pay VAT thereon c. Be effectively subject to zero percent d. Be considered erroneous transaction and must be disregarded. 22. Which of the following franchise grantees is subject to the 2% percentage tax on franchise? a. Franchise on radio and/or television broadcasting companies the gross annual receipts in the preceding year do not exceed P10,000,000. b. Franchise on gas and water utilities. c. Franchise on toll road operations. d. PAGCOR and its licenses and franchisees. 23. Which of the following is subject to value-added tax? a. Medical, dental, hospital, and veterinary services. b. Laboratory services. c. Sale of drugs and medicines in the hospital’s pharmacy or drug store. d. Medicine administered to a patient confined in the hospital. 24. Which of the following statements is incorrect regarding standard deduction? a. A deduction in the amount of P1,000,000 shall be allowed as an additional deduction without need of substantiation. b. The full amount of P1,000,000 shall be allowed as deduction for the benefit of the decedent. c. Standard deduction is a deduction from the conjugal or community properties of married decedents. d. Standard deduction is not allowed to decedents who are nonresidents alien. 25. The actual residential home of the decedent and his family at the time of his death, as certified by the Barangay Captain of the locality where the family home is situated has a fair market value of P1,500,000. The family home is part of the common property of the spouses. How much is the family home deduction? a. a. P1,500,000 c. P750,000 b. b. P1,000,000 d. P500,000

26. Who makes revenue regulations? a. Secretary of Finance b. Commissioner of BIR

c. Board of Accountancy d. House of Reperesentatives

27. Diety is non-stock, non-profit organization made an importation of agricultural product in its original state from a Chinese farmer amounting to P2,240,000. If you are the customs collector how will you treat such importation, will it be? a. a. Subject to 12% VAT c. Exemption from VAT b. Subject to zero rated VAT d. Subject to 3% percentage tax 28. George and Pearl were legally separated. They had six minor children, all qualified to be claimed as additional exemptions for income tax purposes. The court awarded custody of two of the children to George and three to Pearl, with George directed to provide full financial support for them as well. The court awarded the 6th child to George’s father with George also providing full financial support. Assuming that only George is gainfully employed while Pearl is not, how much could George claim as additional exemptions when he files his income tax return? c. P50,000 a. a. P100,000 b. b. P75,000 d. None 29. Construction by XYZ Construction Co. of concrete barrier for the Asian Development Bank in Ortigas Center to prevent car bombs from ramming the ADB gates along ADB Avenue in Mandaluyong City is subject to what business tax? a. a. 12% VAT c. Exemption from VAT d. None of the above b. b. 0% VAT 30. ______________ a number of inquiries from shareholders, James Hong has issued a formal announcement that his company is doing well. a. Following b. Beside c. Against d. Toward 31. Mr. Araki, a non-resident alien stockholder, receives a dividend income of P300,000 in 2012 from a foreign corporation doing business in the Philippines. The gross income of the foreign corporation from within and without the Philippines for three years preceding 2012 are as follows: 2009 From w/in the Philippines P16,000,000 From without the Philippines 18,000,000

2010 P12,000,000 14,000,000

2012 P14,000,000 16,000,000

How much of the dividend income received by Mr. Araki is considered income from sources within the Philippines? a. Zero b. P150,000 c. P270,000 d. P300,000

32. Martin Brian is an independent travelling salesman, deriving his income solely from commissions and personally bearing all expenses without reimbursement or any kind. During the taxable year 2016, Martin Brian incurs the following expenses pertaining directly to his activities as an independent salesman: Travelling expenses Secretarial expenses Long-distance telephone bills Freight expenses for products sold Insurance for product transported Life insurance premiums Doctor’s fee incurred while he was on one of his sales trip

P650,000 250,000 150,000 300,000 50,000 25,000 do not include 15,000 do not include

What amount can Martin Brian deduct from his gross income? a. P1,440,000 b. P1,400,000 c. P1,150,000 d. P40,000 33. A mode of acquisition by virtue of which, the property, rights and obligations, to the extent of the value of the inheritance of a person are transmitted through his death to another either by his will or by operation of law c. Succession a. Extinctive prescription b. Acquisititive prescription d. Donation mortis causa 34. In 2016, Claudio, a self employed resident following: Philippine income tax for 2016 Professional tax Gravel and sand tax U.S.A income tax 2016 Real property tax on his house Road user’s tax on his delivery vans Local annual fixed tax for his delivery vans Interest for late payment of national & local taxes Surcharge for late payment of national & local taxes

citizen,

paid

the

P1,000,000 300 20,000 500,000 3,000 50,000 10,000 40,000 60,000

If Claudio had signified in his return to avail of tax credit of foreign income tax paid, how much deduction may he claim on his 2016 income? a. P1,583,300 c. P580,300 d. P80,300 b. P583,300 35. Rose Company provided fringe benefit to its managerial employees in the amount of P136,000 and to its rank and file employees amounting to P50,000. The deductible expense by Rose Company is: a. P250,000 c. P136,000 b. P186,000 d. P50,000

36. The producers of the new performances because of the shows. a. Overwhelm b. Overwhelming 37. Estate tax is a. An excise tax b. An income tax

play After Dusk decided to add more ___________ positive response to early c. Overwhelmed d. Overwhelmingly c. A percentage tax d. A sales tax

38. The head priest of the religious sect Tres Personas Solo Dios as the corporation sole, rented out 5,000 sq.m. lot registered in its name for use as school site of a school organized for profit. The sect used the rentals for the support and upkeep of its priests. The rented lot is a. Not exempt from real property taxes because the user is organized for profit. b. Exempt from real property taxes since it is actually, directly and exclusively used for religious purposes. c. Not exempt from real property taxes since it is the rents not the land that it used for religious purposes. d. Exempt from real property taxes since it is actually, directly and exclusively used for educational purposes. 39. Which of the following compensation of minimum wage earner shall be taxable? a. Holiday pay c. Night shift differential pay b. Overtime pay d. Commission pay 40. Which principle of sound tax system suggests that the sources of revenue should be sufficient to meet the requirements of the government expenditures? c. Theoretical justice a. Fiscal adequacy b. Administrative feasibility d. Uniformity of taxes 41. The following are the powers of the Bureau of Internal Revenue, except: a. The assessment and collection of all national internal revenue taxes. b. The execution of all judgments in all cases decided in its favor by CTA and the ordinary courts. c. The enforcement of all forfeitures, penalties, and fines connected with national revenue taxes. d. The assessment and collection of customs duties and tariffs. 42. Statement 1 An individual taxpayer is allowed to claim an optional standard deduction of 40% of gross income. Statement 2 A corporate taxpayer is allowed to claim optional standard deduction of 40% of gross income. a. a. True, true b. True, false c. False, true d. False, false

43. Vivian is a VAT registered grocery owner and sugar dealer. She submitted lists of inventory as of December 31, 2016 to the Revenue District Officer as follows: Grocery items, total value P325,000 Raw cane sugar, total value 255,000 In January 2017, she had the following sales and purchases: Sales Purchases Grocery (total invoice value) P1,042,525 P 650,100 Raw cane sugar (excluding VAT) 480,000 420,000 The VAT due for January 2017 is a. P6,675.26 b. P35.675.00

c. P41,675.31

d. P42,045.54

**(1,042,525 X 12%/1.12) - (650,100 X 12%/1.12) = 42,045,54 44. Which is correct? a. Basic personal exemption of a single individual is P20,000. b. Basic personal exemption of a head of the family is P25,000. c. Basic personal exemption of married individual is P32,000. d. Additional personal exemption per qualified dependent P25,000.

is

45. This shall be considered a livelihood activity only and shall not be considered doing business, if the gross sales or receipts in one year does not exceed a. P100,000 c. P200,000 b. P550,000 d. P60,000 46. Antoine Bee is the operator of Globe Bee Coliseum. During the month, it had the following gross receipts from various activities, to wit: I. Concert by April Boy Ireneo P85,000 II. Professional Basketball game 120,000 III. Amateur basketball games 36,700 IV. Billiards 45,000 The percentage tax payable by Antoine Bee on the Coliseum is: a. P18,000 c. P24,750 b. P23,505 d. P43,005 120,000 x 15% = 18,000 47. As franchise. Ms. L.A. Young had the following data on revenues and receivables, taxes not included: Revenue A/R, beginning A/R, ending From operations: Covered by franchise P2,000,000 P300,000 P400,000 Not covered by the franchise 600,000 50,000 How much is the percentage tax, if operating a water system? a. P38,000 b. P52,000 c. P90,000 d. P190,000 300,000 + 2,000,000 - 400,000 = 1,900,000 x 2% = 38,000

Items 48 – 52 are based on the following information: The decedent, a married man with surviving spouse under conjugal partnership of gains, with the following data died on March 1, 2016. Real Properties: Family lot acquired by the decedent before marriage, FMV

P

500,000

Family house built using the salary earned by the surviving spouse during the marriage, FMV

900,000

Coconut farm inherited by the surviving spouse during the marriage, FMV

100,000

Mango orchard, FMV

800,000

Personal Properties: Bank deposit under the name of the decedent representing salary earned before marriage

P2,150,000

Gold necklace inherited by the surviving spouse during the marriage

120,000

Several pieces of jewelry acquired during the marriage using the exclusive money of the decedent

300,000

The following expenses/deductions are claimed: Funeral expenses (40% shouldered by relatives) Judicial expenses (including P50,000 lawyer’s fee incurred on September 5, 2016) A piece of jewelry lost through theft on August 15, 2016 Gambling debts of the decedent Unpaid realty tax (coconut farm) for the 3 quarters of 2016) Claims against an insolvent debtor (25% can be collected) Medical expenses

100,000 120,000 50,000 70,000 150,000 80,000 600,000

48. How much is the conjugal properties? a. P1,780,000 b. P1,700,000 c. P900,000 d. None of the choices 49. How much is the exclusive properties? a. P2,950,000 b. P650,000

c. P450,000 d. None of the choices

50. How much is the family home deduction? a. P1,000,000 b. P950,000

c. P700,000 d. None of the choices

51. How much is the total ordinary deductions? a. P460,000 c. P260,000 b. P310,000 d. None of the choices

Funeral Expense(100,000 x 60%) Judicial Expenses Jewelry theft Claims against Insolvent person Total Ordinary Deductions 52. How much is the taxable net estate? a. a. P1,225,000 b. b. P1,175,000

60,000 120,000 50,000 80,000 310,000 c. P1,125,000 d. None of the choices

Items 53 - 56 are based on the following information: Jon Asty, single resident citizen (who got married during the year 2016), has the following dependents for the taxable year 2016. Leo, adopted child, 25 years old, person with disability, dies during the year Dax Niel, child of his wife in a previous marriage, 18 years old John Denver, acknowledged natural child, becomes 21 years on January 1, 2016 John Rey, child by natural adoption, 15 years old Efren, born December 31, 2016 (taxpayer and his wife’s first born child) The following information pertain to his income and expenses in the year 2016: Salary, gross of P8,500, withholding tax P220,000 Rent expense, apartment house 36,000 Health insurance premium paid (P500/month starting August) 2,500 Gross sales from trading business 150,000 Cost of sales 120,000 Itemized business expenses 10,000 53. The total personal exemptions shall be: a. a. P150,000 b. b. P100,000

c. P75,000 d. P60,000

54. The deductible health insurance premium paid is: a. a. P2,400 c. P1,000 b. b. P1,200 d. None 55. The taxable net income using itemized deduction is: a. a. P164,000 c. P189,000 b. b. P160,000 d. P89,000 56. The taxable net income using optional standard deduction is: a. P164,000 c. P189,000 b. P160,000 d. P89,000

Items 57 - 61 are based on the following information: Joefrey Ochoa Manufacturing Corp. is a VAT-registered enterprise which is also engaged in VAT-exempt transactions. It has the following data taken from its books of accounts for the first quarter of the fiscal year (May 1, 2016 – April 30, 2017) Domestic sale of goods P5,000,000 VAT-Exempt sale of goods 1,000,000 Domestic purchases of goods for use in VAT-subject transactions 500,000 Importation of goods for use in VAT-subject transactions 800,000 Domestic purchases of services for all transactions 450,000 Purchase of vehicles for land transport for use in all transactions (estimated life 5 years, purchased May 1, 2016) 2,400,000 Maintenance expense for the vehicle for land transport 100,000 Other operating expenses 500,000 Monthly VAT paid (May & June) 300,000 57. How much is the creditable input tax? a. a. P460,000 b. b. P223,000

c. P201,000 d. None of the choices

58. How much is the VAT payable? a. a. P140,000 b. b. P377,000

c. P387,000 d. P99,000

59. How much is the taxable net income? a. P3,741,000 b. P3,530,000

c. P3,170,000 d. P2,530,000

60. When shall the VAT return be filed for the first fiscal quarter? a. August 15, 2016 c. August 25, 2016 b. August 20, 2016 d. September 29, 2016 61. When shall the income tax quarter? a. August 25, 2016 b. September 29, 2016

return

be

filed

for

the

first

fiscal

c. September 30, 2016 d. None of the choices

Items 62 - 64 are based on the following information: Jason Elcano Corporation has a soft spot for senior citizens and persons with disability (PWDs). As such, it hires senior citizens and PWDs to work in the company for at least six (6) months. The following data taken from the books of accounts that are provided by the Corporation: Gross sales P15,000,000 Cost of sales 5,000,000 Salaries of senior citizens 500,000 Salaries of PWDs 300,000 Actual amount of assistance under Adopt-A-School program (fully deductible) 200,000 Other operating expenses 3,000,000

During the same period the corporation also allows 20% discount to senior citizens and PWDs who buy goods from the company. The sales are as follows (not part of the gross sales above). Sales to senior citizens P 800,000 Sales to PWDs 500,000 Total P1,300,000 62. How much is the total discount given? a. a. P260,000 b. b. P160,000 63. How much is the total itemized deductions? a. a. P4,480,000 b. b. P4,435,000

itemized

c. P100,000 d. Answer not given

deductions

including

the

special

c. P4,175,000 d. Answer not given

64. How much is the output VAT of the corporation? a. a. P2,079,456 c. P1,828,000 b. b. P1,848,000 d. None of the choices 65. The proceeds received under a life insurance endowment contract is not considered part of the gross income a. If it is so stated in the life insurance endowment policy b. If the price for the endowment policy was not fully paid c. Where payment is made as a result of the death of the insured d. Where the beneficiary was not the one who took out the endowment contract 66. When shall the fringe benefits tax return be filed? a. On or before 10th day of the month following the quarter in which the withholding was made. b. On or before 10th day of the month following the withholding was made. c. On or before 15th day of the month following the quarter in which the withholding was made. d. On or before 15th day of the month following the withholding was made. 67. A, an individual, sold to B, his brother-in-law, his residential lot with market value of P1,000,000 for P600,000. A’s cost in the lot is P100,000. B is financially capable of buying the lot. What tax should be imposed and collected from A as a result of the transaction? a. Presumed capital gains tax c. Real property tax b. Donor’s tax d. Tax on the transfer of real property 68. Which of the following may not raise money for the government? a. Power of taxation c. Power of eminent domain b. Police power d. Privatization of government’s capital assets

69. Riyadh Motors has succeeded _____ generating positive publicity for its new line of ecologically friendly automobiles. a. On b. In c. At d. Of 70. The auditor has _____________ to warrant a full blown forensic audit. a. Much evidence b. b. Many evidence c. c. Much evidences d. d. Many evidences

Related Documents

Taxation Quizzer
February 2021 0
Income Taxation Quizzer
January 2021 6
Quizzer
January 2021 26
Tax Quizzer
February 2021 2
Taxation
January 2021 2
Vat Quizzer
January 2021 1

More Documents from "Anonymous 2ajCCT03VM"

Taxation Quizzer
February 2021 0